Вы находитесь на странице: 1из 61

Problems and Solutions for Abstract Algebra

John A. Beachy & William D. Blair


April 24, 2012
Contents
5 Commutative Rings 3
5.1 Commutative Rings; Integral Domains . . . . . . . . . . . . . . . 3
5.2 Ring Homomorphisms . . . . . . . . . . . . . . . . . . . . . . . . 11
5.3 Ideals and Factor Rings . . . . . . . . . . . . . . . . . . . . . . . 17
5.4 Quotient Fields . . . . . . . . . . . . . . . . . . . . . . . . . . . . 27

6 Fields 32
6.1 Algebraic Elements . . . . . . . . . . . . . . . . . . . . . . . . . . 32
6.2 Finite and Algebraic Extensions . . . . . . . . . . . . . . . . . . . 36
6.3 Geometric Constructions . . . . . . . . . . . . . . . . . . . . . . . 38
6.4 Splitting Fields . . . . . . . . . . . . . . . . . . . . . . . . . . . . 40
6.5 Finite Fields . . . . . . . . . . . . . . . . . . . . . . . . . . . . . . 45
6.6 Irreducible Polynomials over Finite Fields . . . . . . . . . . . . . 48
6.7 Quadratic Reciprocity . . . . . . . . . . . . . . . . . . . . . . . . 53

7 Structure of Groups 56
7.1 Isomorphism Theorems: Automorphisms . . . . . . . . . . . . . . 56
7.2 Conjugacy . . . . . . . . . . . . . . . . . . . . . . . . . . . . . . . 56

1
CONTENTS 2

Here's some random crap from chapter 4 that I did by accident.

9. Let f (x) = an xn + an1 xn1 + a1 x + a0 be a polynomial with rational


coecients. Show that if c is a root of f (x) and c 6= 0, then 1/c is a root of
g(x) = a0 xn + a1 xn1 + + an1 x + an .
Pn a
Examine g(1/c) = j=0 nj cj . Now, if this sum is equal to zero, then so
n
P n anj n
Pn anj Pn
must the sum c j=0 cj . Now c j=0 cj = j=0 anj cnj = f (c) = 0.
n
We now see that g(1/c) = f (c)/c . Since f (c) = 0 and c 6= 0, we conclude that
g(1/c) = 0.

10. Let f (x) = an xn + an1 xn1 + a1 x + a0 be a polynomial with rational


coecients. Show that if c is a root of f (x), and k is a nonzero constant, then
kc is a root of g(x) = bn xn + bn1 xn1 + + b1 x + b0 where bi = k ni ai .
Pn Pn
Examine g(kc) = j=0k nj k j cj = k n j=0 aj cj . Since f (c) = 0, we know
Pn j n
Pn j
that j=0 aj c = 0. Thus, k j=0 aj c = g(kc) = 0. Thus, kc is a root of
g(x).
Chapter 5

Commutative Rings
5.1 Commutative Rings; Integral Domains

1. Which of the following sets are subrings of the eld Q of rational num-
bers? Assume that m , n are integers with n 6= 0 and (m, n) = 1.
m

(a)
m n |n is odd
(b)
 mn |n is even
(c)
 nm|4 - n
n |(n, k) = 1 where k is a xed positive integer

(d)

(a) Since we are very familiar withQ, we know that + is abelian and associa-
tive and we know inverses and identity for +. We also know that distributes
over +. Thus, we only need to show closure. Observe that for a, b, c, d Z,
a c ad+bc
b + d = bd . Now, since b and d are both odd (they do not have a factor of
2), there is no way that bd can be even. (Even if there is some cancellation that
occurs, a factor of 2 can never magically appear in the denominator.)
1 1 1
/ m

(b) Examine
2 + 2 = 1 = 1 n |n is even . Thus, the group property is
not satised, so (b) is not a subring.
1 2 13
/ m

(c) Examine
3 + 7 = 28 n |4 - n . Thus, the group property is not
satised, so (b) is not a subring.
a c ad+bc
(d) Again we only need to show closure. since
b + d := bd , and neither b
nor d share any factor in common with k , it is clear that (bd, k) = 1. Thus, (d)
is a subring.

2. Which of thefollowing sets are subrings of the eld R of real numbers?


(a) A = {m + n 2|m, n Z and n is even}
(b) B = {m + n 2|m, n Z and m is odd}
3
(c) C = {a + b 2|a, b Q}
3

3
(d) D = {a + b 2 + c 9|a, b, c Q}

(e) E = {m + nu|m, n Z},where u = (1 + 3)/2

3
CHAPTER 5. COMMUTATIVE RINGS 4


(f ) F = {m + nv|m, n Z}, where v = (1 + 5)/2

(a) As in Exercise 1, we will only need to show closure of + (or lack thereof ).

We see that (a + b 2) + (c + d 2) = (a + c) + (b + d) 2. The sum of two even
numbers is even, so A is a subring.

(b) Examine (3 + 2) + (3
+ 2) = 6 + 2 2/ B . Thus, B is not
a subring.
3 3 3
(c) Similar to (a), (a + b 2) + (c + d 2) = (a + c) + (b + d) 2. The sum
of two rationals is rational, so C is a subring.
(d) Similar to (c), D is a subring.

1+ 3

(e) Examine (1 + u) + (1 + u) = 2 + 2u = 2 + 2 = 3+ 3 / E . Thus,
2
E is not a subring.
(f ) Similarly, F is not a subring.
(Note: Sets like A, B, C, D, E, and F can almost be thought of as vector
spaces. For example, D can be thought of as a vector space over Q with basis

{1, 3 2, 3 9}. Then, one only needs to check that the set from which the co-
ecients come is closed, since one can always write vectors as a unique linear
combination. Sets like A don't technically form a vector space since Z is a ring
and not a eld. However, there is still a concept of basis associated with this
set. Such a set with a basis is called a free module.)

 3. Consider which of the following conditions on the set of all 2 2 matrices


a b
with rational entries. Which conditions below dene a commutative
c d
ring? If the set is a ring, nd all units.
(a) all matrices with d = a, c = 0
(b) all matrices with d = a, c = b
(c) all matrices with d = a, c = 2b
(d) all matrices with d = a, c = b
(e) all matrices with c = 0
(f ) all matrices with a = 0 and d = 0.

(a) We know that matrix addition is commutative and we know inverses


and identity. Furthermore, matrix multiplication distributes over matrix addi-
tion.
       . Exam-
Thus, we only need to check for closure and commutivity of
  
a b c d a+c b+d a b c d
ine + = . Also, =
 0 a 0 c 0 a+c 0 a 0 c
    
ac ad + bc c d a b
= . Thus, (a) denes a commutative ring.
0 ac 0 c 0 a
 1  1 b 
a b a a2
From elementary linear algebra, we see that = which
0 a 0 a1
is dened so long as a 6= 0. So, the set of units is the set dened by (a) minus
any matrix where a = 0.
       
a b c d a+c b+d a b
(b) Examine + = . Also,
b a d c b+d a+c b a
CHAPTER 5. COMMUTATIVE RINGS 5

      
c d ac + bd ad + bc a b c d
= = . Thus, (b) denes a
d c ad + bc ac + bd b a d c
 1  a b 
a b a2 b2 a 2 b2
commutative ring. From elementary linear algebra, = b a
b a a2 b2 a2 b2
2 2
which is dened as long as a b 6= 0. Thus, the set of units is the set dened
2 2
by (b) minus those elements where a b = 0.
     
a b c d a+c b+d
(c) Examine + = . Also,
  2b  a  2d c 2(b
 +  d) a + c  
a b c d ac 2bd ad + bc c d a b
= = .
2b a 2d c 2ad 2bc ac 2bd 2d c 2b a
 1  
a b 1 a b
Thus, (c) denes a commutative ring. Now, = a2 +2b 2
2b a 2b a
which is dened as long as a and b are not both equal to 0.
     
a b c d a+c b+d
(d) Examine + = . Multiplica-
b a d c (b + d) a + c
1
tion is commutative since we can factor out
2 andobtain elements in (c). Thus,
 1  
a b 1 a b
(d) denes a commutative ring. We see = a2 +b2 which
b a b a
again is dened as long as a and
b are not both 0.
     
3 2 4 5 12 27 4 5
(e) Closure is clear. However, = 6=
0 1 0 6 0 6 0 6
   
3 2 12 13
= . Thus, (e) does not dene a commutative ring.
0 1 0 6
       
0 1 0 3 4 0 0 3
(f ) Closure is clear. However, = 6=
    2 0 4 0 0 6 4 0
0 1 6 0
= . Thus, (f ) does not dene a commutative ring.
2 0 0 4

4. Let R = {m + n 2|m, n Z}.
(a) Show that m + n 2 is a unit in R if and only if m 2n = 1.
2 2

(b) Show that 1 + 2 2 has innite order in R .

(c) Show that 1 and 1 are the only units that have nite order in R .


(a) We see that m + n 2is a unit if and only if there is an x R such

that x(m + n 2) = (m + n 2) = 1. By commutivity of , we only need to
1
consider x(m + n 2). Now, x(m + n 2) = 1 if and only if x = .
m+n 2
Multiplying numerator and denominator by the conjugate, this implies x =
1
m n
m2 2n2 (m n 2). For x to be a member of R, we need m2 2n2 , m2 2n2 Z.
2 2
Thus, x R if and only if m 2n = 1.
r Pr s
 
r
(b) By the binomial theorem, we have that (1 + 2 2) = s=0 s (2 2)

which clearly can become arbitrarily large. Thus, o(1 + 2 2) = .

(c) First of all, by (a), any unit must have the form 1 + 2n2 + n 2.
r Pr s rs
Then, by the binomial theorem, 1 + 2n2 + n 2 = s=0 1 + 2n2 nrs 2 .
CHAPTER 5. COMMUTATIVE RINGS 6


This can become arbitrarily large unless
n=0 in which case 1 + 2n2 +
n 2 = 1.

5. Let R be a subset of an integral domain D. Prove that if R is a ring


under the operations of D, then R is a subring of D.

Closure for + and is assumed since R is a ring under the operations of D.


Since (R, +) is an abelian group, we must have inverses. Thus, if a R, we
must have a R. Finally, suppose that x R. Then, since 1 x = x R, we
must have 1 R by closure. Thus, R is a subring of D .

6. Let D be a nite integral domain. Give another proof of Theorem 5.1.9


by showing that if d is a nonzero element of D, then d1 = dk for some positive
integer k. [Theorem 5.1.9 states that any nite integral domain is a eld.]

d D. By closure, d d D. Now, D is assumed to be nite, so for


Let
some m and n, we must have dm = dn . (Otherwise D would have to be innite.)
m
Then, d = dn 1 = dnm by the cancellation property for integral domains.
nm1 1
Thus, 1 = d d . Set k = n m 1. Then clearly d = dk . Thus, every
element has an inverse, so D is a eld.

7. An element a of a commutative ring R is called nilpotent if an = 0 for


some positive integer n. Prove that if u is a unit in R and a is nilpotent, then
u a is a unit in R.

Consider the element an 1 = 1. Now, 1 is clearly a root of an 1, so we


n
can write a 1 = (a 1) p(a) where p(a) is a polynomial in the indeterminant
a. Thus, (a 1) p(a) = 1, so (a 1) is a unit. (We see that p(a) R since it
is just linear combos of powers ofa.)
1
Now, let u R . Then, (a u) = u(au 1). Since a is nilpotent, so is
au . Thus, by the previous paragraph, (au1 1) R . Since u R , we
1
1
must have u(au 1) R .

8. Let R be a commutative ring such that a2 = a for all a R. Show that


a + a = 0 for all a R.

Notice that(a+1)2 = a2 +2a+1. But, a2 = a for all a R, so (a+1)2 = a+1


and a + 1 = a + 2a + 1. Thus, a + 1 = 3a + 1 implying 0 = 2a, as desired.

9. Let I be any set and let R be the collection of all subsets of I . Dene
addition and multiplication of subsets A, B I as follows:
A + B = (A B) A B and A B = A B.

Show that R is a commutative ring under this addition and multiplication.


CHAPTER 5. COMMUTATIVE RINGS 7

We simply verify the axioms.


(Closure of +) Since A, B I , A B I by denition of union. Then
the intersection of any set with a set in I is completely contained in I . Thus,
A + B I.
(Associativity of +) Omitted.
(Zero Element) is the zero element: A + = (A ) ( A) = A .
(Additive Inverses) If A R, then A + A = (A A) (A A) = . Thus,
every element is its own additive inverse.
(Multiplicative identity) I is the multiplicative inverse: AI = AI =
I A = A.
(Commutivity) Given since is commutative.
(Distributivity of over +) Omitted.

10. For the ring R dened in Exercise 9, write out addition and multiplica-
tion tables for the following cases:
(a) I has two elements;
(b) I has three elements.

(a) Suppose I = {a, b}. Then 2I = {I, {a}, {b}, }. Here is the addition
table:

Here is the multiplication table:

(b) Suppose I = {a, b, c}. Then 2I = {I, {a, b}, {a, c}, {b, c}, {a}, {b}, {c}, }.
Here is the addition table:

Here is the multiplication table:


CHAPTER 5. COMMUTATIVE RINGS 8

(b) Omitted

11. A commutative ring R is called a Boolean ring if a2 = a for all a R.


Show that in a Boolean ring the commutative law follows from the other axioms.

Let a, b R. Then ab = ba a2 b = aba ab = aba ab2 = abab ab =


2
(ab) . Since R is boolean, the last equality is true, so all the equalities that are
derived from it including ab = ba are true. This completes the proof.

12. Let I be any set and let R be the collection of all subsets of I . Dene
addition and multiplication of subsets A, B I as follows:
A + B = A B and A B = A B.

Is R a commutative ring under this addition and multiplication?

Observe that is the additive identity of R since A = A for all A I.


However, one can never union a set to any nonempty set and obtain . Thus,
unless I = , R is not a ring.

13. Let R be the set of all continuous functions from the set of real numbers
to itself.
(a) Show that R is a commutative ring if the formulas (f +g)(x) = f (x)+g(x)
and (f g)(x) = f (x) g(x) for all x R are used to dene addition and
multiplication of functions.
(b) Which properties in the denition of commutative ring fail if the product
of two functions is dened to be (f g)(x) = f (g(x)) for all x?

(a) (Associativity of +) Given since + is associative.


(Additive Identity) The function f 0 is continuous and (g + f )(x) = g(x)
for all continuous functions g.
(Additive Inverses) For a continuous function f , the function f (x) = (1)
f (x) is its additive inverse.
(Commutativity) Given since is commutative.
(Distributivity) (f (g +h))(x) = f (x)[g(x)+h(x)] = f (x)g(x)+f (x)h(x) =
(f g)(x) + (f h)(x).

f (x) = x2
(b) Commutativity fails and so does distributivity. For example, if
2 2 2
and g(x) = x, then (f g)(x) = x , but (gf ) = x . Also, if f (x) = x , g(x) =
3 4 3 4 2
x , and h(x) = x , then f (g + h)(x) = (x + x ) , while f (g(x)) + f (h(x)) =
x6 + x8 .

14. Dene new operations on Q by letting a b = a + b and a b = 2ab for


all a, b Q. Show that Q is a commutative ring under these operations.

(Group under +) Given.


CHAPTER 5. COMMUTATIVE RINGS 9

(Commutativity of ) a b = 2ab = 2ba = b a.


(Distributivity of over ) a (bc) = 2a(b+c) = 2ab+2ac = (a b)(a c)

15. Dene new operations on Z by letting m n = m + n 1 and m n =


m + n mn, for all m, n Z. Is Z a commutative ring under these operations?

(Associativity of ) Given by associativity of + and -.


m + 1 1 = m for all m Z.
(Zero element) 1 is the additive identity since
m Z, m+2 is such that m+(m+2)1 = 1.
(Additive Inverseses) For all
(Identity element) 0 is the identity element since m 0 = m + 0 m(0) = m.
(Commutativity of ) m n = m + n mn = n + m nm = n m.
(Distributivity of over ) ` (m n) = ` (m + n 1) = ` + m + n
1 (`m + `n `) = (` + m `m) + (` + n `n) 1 = (` m) (` n).

16. Let R and S be commutative rings. Prove that the set of all ordered
pairs (r, s) such that r R and s S can be given a ring structure by dening
(r1 , s1 ) + (r2 , s2 ) = (r1 + r2 , s1 + s2 ) and (r1 , s1 ) (r2 , s2 ) = (r1 r2 , s1 s2 ).

This is called the direct sum of R and S , denoted by R S .

(Associativity of +) Given by the associativity of vector addition.


(Zero Element) The element (0R , 0S ) where 0R and 0S are the zero elements
of R and S R S.
respectively is the zero element of
(Additive Inverses) If (a, b) R S , then (a, b) is its inverse since (a, b)+
(a, b) = (0R , 0S ).
(Identity Element) The element (1R , 1S ) where 1R and 1S are the identity
elements of R and S is the identity element since (a, b) (1R , 1S ) = (a, b).
(Commutativity) (a, b) (c, d) = (ac, bd) = (ca, db) = (c, d) (a, b).
(Distributivity) (a, b)[(c, d)+(e, f )] = (a, b)(c+e, d+f ) = (ac+ae, bd+bf ) =
(a, b) (c, d) + (a, b) (e, f ).
Thus, R S is a commutative ring.

17. Give addition and multiplication tables for Z2 Z2 .

Omitted.

18. Generalizing to allow the direct sum of three commutative rings, give
addition and multiplication tables for Z2 Z2 Z2 .

Omitted.

19. Find all the units of the following rings.


(a) ZZ
(b) Z4 Z9
CHAPTER 5. COMMUTATIVE RINGS 10

(a) From Q16, the identity element is (1, 1). Now, (a, b) (c, d) = (1, 1)
(ac, bd) = (1, 1). Since a, b, c, d Z, we can only have ac = 1 and bd = 1 if
a = b = c = d = 1.
(b) From Q16, the identity element is ([1]4 , [1]9 ). Now, ([a]4 , [b]9 )([c]4 , [d]9 ) =
([1]4 , [1]9 ) ([ac]4 , [bd]9 ). Now, we simply need to nd the units of Z4 and
Z9 . We know Z 4 = {[1]4 , [3]4 } and Z9 = {[1]9 , [2]9 , [4]9 , [5]9 , [7]9 , [8]9 }. Thus,
(Z4 Z9 ) = {(x, y)|x Z4 and y Z9 }.

20. An element e of a ring R is said to be idempotent if e2 = e. Find all


idempotent elements of the following rings.
(a) Z8 and Z9
(b) Z10 and Z12
(c) ZZ
(d) Z10 Z12

Z8 : [0]8 , [1]8
(a) Idempotent elements of
Z9 : [0]9 , [1]9
Idempotent elements of
(b) Idempotent elements of Z10 : [0]10 , [1]10 , [5]10 , [6]10
Idempotent elements of Z12 : [0]12 , [1]12 , [4]12 , [9]12
(c) Idempotent elements of Z Z: (0, 0), (1, 1)
(d) Idempotent elements of Z10 Z12 : {(x, y)|x {idempotent elements of Z10 }, y
{idempotent elements of Z12 }}

21. Let A be an abelian group, and let R = {(a, b)|a A and n Z}.
Dene binary operations + and on R by (a, n) + (b, m) = (a + b, n + m) and
(a, n) (b, m) = (am + bn, nm), for all (a, n) and (b, m) in R. Show that R is a
commutative ring.

(Group property) Satised since the underlying group structure is the carte-
sian product of two groups.
(Identity element) We need an element (x, y) R such that (x, y) (a, n) =
(xn + ay, n) = (a, n). (x, y) (a, n) = (a, n), we must have yn = n
In order for
which can only happen if y = 1. Thus, we need xn + a = a which implies that
x = 0A . Thus, (0A , 1) is the identity element.
(Commutativity) Given.
(Distributivity) (a, `)[(b, m)+(c, n)] = (a, `)(b+c, m+n) = ((a(m + n) + `(b + c), `(m + n)).
(1)
(a, `) (b, m) + (a, `)(c, n) = (am + `b, `m) + (an + `c, `n) = (am + `b + an +
`c, `m + `n) = (a(m + n) + `(b + c), `(m + n)). (2)
We see that (1) = (2), so distributivity has been shown.

22. Let R be a set that satises all the axioms of a commutative ring, with
the exception of the existence of a multiplicative identity element. Dene binary
operations + and on R1 = {(r, n)|r R, n Z} by (r, n) + (s, m) = (r +
s, n + m) and (r, n) (s, m) = (rs + ns + mr, nm), for all (r, n) and (s, m) in R1 .
Show that R1 is a commutative ring with identity (0, 1) and that {(r, 0)|r R}
CHAPTER 5. COMMUTATIVE RINGS 11

satises all the conditions of a subring, with the exception that it does not have
the multiplicative identity of R.

(Group property) Satised since the underlying group structure is the carte-
sian product of two groups.
(Identity element) Similar to Q21.
(Commutativity) Given except for multiplication with the identity element.
Since we showed that (0,1) was the identity element, we know it commutes (by
denition of identity element).
(Distributivity) (a, `)[(b, m)+(c, n)] = (a, `)(b+c, n+m) = (a(b + c) + `(b + c) + a(n + m), `(n + m)).
(1)
(a, `) (b, m) + (a, `) (c, n) = (ab + `b + am, `m) + (ac + `c + an, `n) =
(ab+`b+am+ac+`c+an, `m+`n) = (a(b + c) + `(b + c) + a(m + n), `(m + n)).
(2)
We see (1) = (2), so distributivity has been shown.

5.2 Ring Homomorphisms

1. Let R be a commutative ring and let D be an integral domain. Let


: R D be a nonzero function such that (a + b) = (a) + (b) and (ab) =
(a)(b), for all a, b R. Show that is a ring homomorphism.

We must simply show that (1) = 1. Now, (a) = (1 a) = (1) (a).


Since D is an integral domain, the cancellation property can be used to obtain
(a) = (1) (a) 1 = (1).

2. Let F be a eld and let : F R be a ring homomorphism. Show that


is either zero or one-to-one.

Clearly 0 is a possible mapping.


Suppose that (a) = (b) for some a, b F . Since F is a eld, this implies
that (ab1 ) = 1. This then implies that ab1 = 1 which implies that a = b.

3. Let F, E be elds, and let : F E be a ring homomorphism. Show


that if is onto, then must be an isomorphism.

We only need to show that is one-to-one. Suppose that (x1 ) = (x2 ) = y .


Then, since E is a eld, (x1 ) (x2 )1 = 1. Since F is a eld, we see that this
1 1
implies that (x1 x2 ) = 1. This means that xx2 = 1, so x1 = x2 . Thus, is
one-to-one.

4. Show that taking complex conjugates denes an automorphism of C. That


is, for z C, dene (z) = z , and show that is an automorphism of C.
CHAPTER 5. COMMUTATIVE RINGS 12

(1-1) Let z = rei . Then (rei ) = rei . This mapping is one-to-one then
by the uniqueness of additive inverses.
(Onto) Let w = rei . Then, z = rei gives (z) = w.
(Preservation) Let z = a + ib and w = c + id. Then (z + w) = [(a + c) +
i(b + d)] = (a + c) i(b + d) = (a ib) + (c id) = (z) + (w).
Let z = re
i i
and w = se . Then, (zw) = (rse
i(+)
) = rsei(+) =
i i
re se = (z)(w).
(Identity) Satised since C is a eld. (See Q1.)

5. Show that the identity mapping is the only ring homomorphism from Z
into Z.

: Z Z be a ring homomorphism. Then, we must have (1) = 1.


Let
Since for any n Z, n = 1 + 1 + + 1 (n terms), we must have (n) =
(1 + 1 + + 1) = (1) + (1) + + (1) = 1 + 1 + + 1 = n.
 
m n
6. Show that the set of all matrices over Z of the form is a ring
2n m

isomorphic to the ring Z[ 2] dened in Example 5.1.6.

Let S be the set described in the statement of the problem. Dene :



 
a b
Z[ 2] S via (a + b 2) = .
2b m
(1-1) This map is a direct substitution, so it is 1-1.

   
m n m n
(Onto). Let S . Then, (m + n 2) = .
2n m 2n m
 
1 0
(Identity) (1) = .
0 1

 
a+c b+d
(Preservation) [(a+b 2)+(c+d 2)] = [(a+c)+(b+d) 2] = =
2(b + d) a + c

   
a b c d
+ = (a + b 2) + (c + d 2).
2b a 2d c

 
ac + 2bd ad + bc
[(a+b 2)(c+d 2)] = [(ac+2bd)+(ad+bc) 2] = .
2(ad + bc) ac + 2bd
(1)

     
a b c d ac + 2bd ad + bc
(a + b 2)(c + d 2) = = .
2b a 2d c 2(ad + bc) ac + 2bd
(2)
We see (1) = (2), so preservation has been shown.


7. Dene : Z[ 2] Z[ 2] by (m + n 2) = m n 2, for all m, n Z.
Show that is an automorphism of Z[ 2].

This is similar to Q4. We will only show preservation of products. (The


identity mapping is very easily veried).
CHAPTER 5. COMMUTATIVE RINGS 13


[(a + b 2)(c + d 2)] = [ac + 2bd + (ad + bc) 2] = ac + 2bd (ad + bc) 2.
(1)

(a + b 2)(c + d 2) = (a b 2)(c d 2) = ac + 2bd (ad + bc) 2. (2)
(1) = (2), so preservation is shown.

8. Let F be a eld, and let a F . Dene : F [x] F [x] by (f (x)) =


f (x + a), for all f (x) F [x]. Show that is an automorphism of F .

(1-1) The transformation f (x + a) simply takes the graph of f (x) and shifts
it left a units. (This does make sense in an abstract setting since the denition
of graph is simply the set of ordered pairs of the form (x, f (x)).) Thus, this
mapping of f (x) 7 f (x + a) is a direct substitution and thus one-to-one.
(Onto) Let f (x) F [x]. Then, f (x a) 7 f (x).
(Identity) Let f 1. Then, f (x + a) 1.
n n1
(Preservation) Let f (x) = an x + an1 x + + a1 x + a0 and g(x) =
m m1
bm x + bm1 x + + b1 x + b0 . Without loss of generality, suppose deg g
deg f .
[f (x)+g(x)] = [bm xm +bm1 xm1 + +(an +bn )xn +(an1 +bn1 )xn1 +
+ (a1 + b1 )x + (a0 + b0 )]= bm (x + a)m + + (an + bn )(x + a)n + + a0 + b0 =
(bm (x + a)m + + b0 ) + (an (x + a)n + + a0 ) = (g(x)) + (f (x)).
Let f (x) = (xr1 )(xr2 ) (xrn ) and g(x) = (xs1 )(xs2 ) (xsm ).
[f (x)g(x)] = [(xr1 ) (xrn )(xs1 ) (xsm )] = (x+ar1 ) (x+
a rn )(x + a s1 ) (x + a sm ) = (f (x))(g(x)).

9. Show that the composition of two ring homomorphisms is a ring homo-


morphism.

Let : S T and : R S be ring homomorphisms. Dene : R T


via (x) = ((x)) for all x R.
(Preservation of +) ((x + y)) = ((x) + (y)) = ((x)) + ((y)).
(Preservation of ) ((xy)) = ((x)(y)) = ((x))((y)).
(Identity) ((1)) = (1) = 1.

10. Let R and S be rings, and let , : R S be ring homomorphisms.


Show that {r R|(r) = (r)}is a subring of R.

r, s R. Then (r + s) = (r) + (s) = (r) + (s) =


(Closure under +) Let
(r + s). (r + s) R.
Thus,
(Closure under ) (rs) = (r)(s) = (r)(s) = (rs). Thus, rs R.
(Additive inverses) Let r R. Then (r) = (r) = (r) = (r).
Thus, r R.
(Identity) (1) = 1 = (1). Thus, 1 R.

11. Show that the direct sum of two nonzero rings is never an integral
domain.
CHAPTER 5. COMMUTATIVE RINGS 14

Let R and S be nonzero rings. Then, we always have (1, 0) (0, 1) = (0, 0).
Neither element is zero, but the product is the zero of R S.

12. Let R1 and R2 be commutative rings.


Dene 1 : R1 R2 R1 by 1 ((r1 , r2 )) = r1 , for all (r1 , r2 ) R1 R2
(a)
and dene 2 : R1 R2 R2 by 2 ((r1 , r2 )) = r2 for all (r1 , r2 ) R1 R2 .
Show that 1 and 2 are ring homomorphisms.
(b) Let R be any ring, and let : R R1 R2 be a function. Show that is
a ring homomorphism if and only if 1 and 2 are both ring homomorphisms.

(a) (Preservation of +) 1 [(r, s) + (v, w)] = 1 (r + v, s + w) = r + v =


1 (r, s) + 1 (v, w).
(Preservation of ) 1 [(r, s) + (v, w)] = 1 (rv, sw) = rv = 1 (r, s)1 (v, w).
(Identity) 1 (1, 1) = 1.
Thus, 1 is a ring homomorphism. Similarly, 2 is a ring homomorphism.

(b) ()Suppose that one of


1 or 2 is not a homomorphism. Without
loss of generality, suppose that1 is not a homomorphism. By Q9, if 1
and are homomorphisms, then 1 is a homomorphism. By contraposition,
if 1 is not a homomorphism, then one of 1 or is not a homomorphism.
Part (a) showed that 1 is a homomorphism, so we conclude that is not a
homomorphism.
() Suppose that is not a ring homomorphism. Then, at least one of
three things must occur: (1) (r + s) 6= (r) + (s), (2) (rs) 6= (r)(s), or
(3) (1) 6= 1.
(r + s) = (v, w) while (r) = (a, b) and (s) = (c, d).
For (1), suppose that
Then, 1 ((v + w)) = 1 ((a + c, b + d)) = a + c. Similarly, 2 ((v + w)) =
b + d. Now, it is possible for either a + c = v or b + d = w, but not both.
For if both held, it would imply that 1 ((v + w)) = 1 ((v) + (w)) and
2 ((v + w)) = 2 ((v) + (w)), which would imply that 1 and 2 are not well
dened. Contradiction. Thus, one of 1 or 2 is not a homomorphism.
(2) Follows from a similar argument.
For (3), if (1) 6= (1, 1), then at least one of 1 (1, 1) or 2 (1, 1) is not equal
to the identity. Thus, one of 1 or 2 is not a homomorphism.

13. Find all ring homomorphisms from Z Z into Z. That is, nd all
possible formulas and show why no others are possible.

: ZZ Z be a homomorphism. Then, since the identities must map


Let
to each other, we see that (1, 1) 7 1. Then, since n = 1 + 1 + + 1 (n terms)
for any n Z, (n, n) = n for all n Z. Thus, we must have (m, n) = m or
(m, n) = n for all (m, n) Z Z.

14. Find all ring homomorphisms from Z Z into Z Z.


CHAPTER 5. COMMUTATIVE RINGS 15


As above, we must have (1, 1) 7 (1, 1). By extension, we must have (n, n) 7
(n, n). Thus, we must have (m, n) = (m, n), (m, n) = (m, m), or (m, n) =
(n, n).

15. For the rings Zn and Zk , show that if k|n, then the function : Zn Zk
dened by ([x]n ) = [x]k , for all [x]n Zn , is a ring homomorphism. Show that
this is the only ring homomorphism from Zn to Zk .

(Preservation of +) ([a]n + [b]n ) = ([a + b]n ) = [a + b]k = [a]k + [b]k =


([a]n ) + ([b]n ).
(Preservation of ) ([a]n [b]n ) = ([ab]n ) = [ab]k = [a]k [b]k = ([a]n )([b]n ).
Now, as mentioned many times before, if : Zn Zk is a homomorphism,
we must have [1]n 7 [1]k . Also, since for any r Z, n = 1 + 1 + + 1 (r
terms), we know that ([r]n ) = ([1]n + [1]n + + [1]n ) = ([1]n ) + ([1]n ) +
+ ([1]n ) = [1]k + [1]k + + [1]k = [r]k . Thus ([x]n ) = [x]k is the only
homomorphism from Zn to Zk .

16. Are Z9 and Z3 Z3 isomorphic as rings?

Let : Z9 Z3 Z3 . If is to be an isomorphism, we need [1]9 7


([1]3 , [1]3 ). However, 3 [1]9 = [3]9 while3 ([1]3 , [1]3 ) = ([0]3 , [0]3 ), so the
underlying additive groups are not isomorphic, so Z9  Z3 Z3 .

17. Let S be the subset of Z4 Z4 given by {([m]4 , [n]4 )|m n(mod 2)}.
(a) Show that S is a subring of Z4 Z4 .
(b) Show that S is not isomorphic (as a ring) to any ring of the form Zn ,
nor to any direct sum of such rings.

(a) (Closure under +) Let ([a]4 , [b]4 ), ([c]4 , [d]4 ) S . Then, ([a]4 , [b]4 ) +
([c]4 , [d]4 ) = ([a + c]4 , [b + d]4 ). Now if [a]2 = [b]2 and [c]2 = [d]2 , then [a + c]2 =
[b + d]2 by simple adding of two valid equations. Thus ([a + c]4 , [b + d]4 ) S .
(Closure under ) Let ([a]4 , [b]4 ), ([c]4 , [d]4 ) S . Then, ([a]4 , [b]4 )([c]4 , [d]4 ) =
([ac]4 , [bd]4 ). Now there are two possibilities: (1) [c]2 = [d]2 = [0]2 , or (2)
[c]2 = [d]2 = [1]2 . So, when we evaluate [ac]2 and [bd]2 , in (1) we get [0]2 and
[0]2 and in (2) we get [a]2 and [b]2 . Thus, ([ac]4 , [bd]4 ) S .
(Additive inverses) If [a]2 = [b]2 , then [a]2 = [b]2 . Thus, if ([a]4 , [b]4 ) S ,
we have ([a]4 , [b]4 ) S .
(Identity Element) Clearly, [1]2 [1]4 , so ([1]4 , [1]4 ) S .
Thus, S is a subring of Z4 Z4 .

(b) We note that the element ([1]4 , [1]4 ) S has multiplicative order 4.
Thus, if S were isomorphic to a direct sum of residue rings, it would have to be
isomorphic to Z4 Z4 . However, ([1]4 , [2]4 ) Z4 Z4 S . Thus S  Z4 .
If S were to be isomorphic to a ring of residues, it would have to be iso-
morphic to Z4 . However, ([n]4 , [n]4 ) are in S for all n, giving four congruence
CHAPTER 5. COMMUTATIVE RINGS 16

classes. In addition, ([1]4 , [3]4 ) S . Thus, |S| > |Z4 |, so S  Z4 .

18. Dene : Z Zm Zn by (x) = ([x]m , [x]n ). Find the kernel and


image of . Show that is onto if and only if gcd(m, n) = 1.

ker = {x Z : n|x and m|x}. The image is {([x]m , [x]n )|x


We see that
Z}. gcd(m, n) = 1, then m and n share no factors, so no integers besides 0
If
and kmn where k Z can be mapped to ([0]m , [0]n ). Since |Zm Zn | = mn,
the rst mn integers will be mapped to unique elements of Zm Zn . Thus,
is onto.
gcd(m, n) = d 6= 1, then we have m = dr and n = dq for some
Conversely, if
r, q Z. Thus,drq 7 ([0]m , [0]n ). Since drq < mn, the mapping will replicate
all the previous (drq1) mappings over and over again without having generated
mn unique elements (the order of Zm Zn ). Thus, is not onto.

19. Let R be the ring given by dening new operations on Z by letting


m n = m + n 1 and m n = m + n mn. Dene : Z R by (n) = 1 n.
Show that is an isomorphism.

is a linear map.
(1-1) Given since
(Onto) Lety R. Then (y + 1) = 1 (y + 1) = y . Thus, is onto.
(Preservation of sums) (m + n) = 1 (m + n) = 1 m n (1).
(m) (n) = (1 m) (1 n) = 1 m + 1 n 1 = 1 m n (2). Now,
(1) = (2), so sums are preserved.
(Preservation of products) (mn) = 1 mn. (1)
(m) (n) = (1 m) (1 n) = 1 m + 1 n (1 m n + mn) = 1 mn
(2). Again, (1) = (2), so products are preserved.

20. Let I be any set and let R be the collection of all subsets of I . Dene
addition and multiplication of subsets A, B I as follows:
A + B = (A B) A B and A B = A B.

(a) Show that if I has two elements then R = Z2 Z2 .


(b) Show that if I has three elements then R
= Z2 Z2 Z2 .

For both of these, one can merely construct multiplication- and addition
tables and note that the rings behave in the same way. The multiplication- and
addition tables for R in these two particualar cases were already constructed in
Q10 of 5.1.

21. Let R1 , R2 , ..., Rn be commutative rings. Complete the proof of Propo-


sition 5.2.8, to show that R = R1 R2 Rn is a commutative ring. Then
show that R = R1 R2 Rn .
CHAPTER 5. COMMUTATIVE RINGS 17

(Group Property) Given by Proposition 3.3.4.


(Commutativity) Given by the commutativity of R1 , ..., Rn .
(Identity Element) (1R1 , 1R2 , ..., 1Rn ) is obviously the identity element.
(Distributivity) (r1 , ..., rn )[(s1 , ..., sn )+(t1 , ..., tn )] = (r1 , ..., rn )(s1 +t1 , ..., sn +
tn ) = (r1 [s1 +t1 ], ..., rn [sn +tn ]) = (r1 s1 +r1 t1 , ..., rn sn +rn tn ) = (r1 , ..., rn )(s1 , ..., sn )+
(r1 , ..., rn )(t1 , ..., tn )

Let u = (u1 , ..., un ) R . We must have uu1 = 1R = (1R1 , ..., 1Rn ). Thus,
(u1 , ..., un )u1 = (1R1 , ..., 1Rn ). We see that because R1 , ..., Rn are commutative

rings, this can only happen if ui Ri and u
1
= (u1 1
1 , ...un ). Thus, R =

R1 R2 Rn .

22. Let R be an integral domain. Show that R contains a subring isomorphic


to Zp for some prime number p if and only if char(R) = p.

char(R) = p p 1 = 0 for some prime p 1 + 1 + + 1 = 0 (p terms)


< 1 >= Zp .

23. Show that if R is an integral domain with characteristic p > 0, then for
all a, b R we must have (a + b)p = ap + bp . Show by induction that we must
also have (a + b) = a + bp for all positive integers n.
pn pn n

 
p pk k Pp
p
Pp p!
By the binomial theorem, (a+b) = k=0 k a b = k=0 k!(pk)! apk bk .
p!
Clearly,
k!(pk)! will always yield a factor of p unless k = 0 or k = p. Thus, all
the terms of the sum are 0 except for the terms where k = 0 or k = p. Thus,
(a + b)p = ap + bp .
We have established a basis for induction. Suppose the statement is true for
N +1 N
p
all positive integers less than or equal to
 p  p  p N. Then,
p (a +Nb) = (a + b)p p =
N N N N N +1 N +1
(a + b)p = ap + bp = ap + bp = ap + bp , as desired.

5.3 Ideals and Factor Rings

1. Give a multiplication table for the ring Z2 [x]/hx2 + 1i.

We see that Z2 [x]/hx2 + 1i = {ax + b|a, b Z2 } = {x + 1, x, 1, 0}. We use


2
the relation x + 1 = 0 to obtain the multiplication table.

0 1 x x+1
0 0 0 0 0
1 0 1 x x+1
x 0 x 1 x+1
x+1 0 x+1 x+1 0
CHAPTER 5. COMMUTATIVE RINGS 18

2. Give a multiplication table for the ring Z2 [x]/hx3 + x2 + x + 1i.

We see that Z2 [x]/hx3 +x2 +x+1i = {ax2 +bx+c|a, b, c Z2 } = {0, 1, x, x+


1, x , x + x, x + 1, x2 + x + 1}. We use the relation x3 + x2 + x + 1 = 0 to
2 2 2

obtain the multiplication table.

0 1 x x+1 x2 x2 + x x2 + 1 x2 + x + 1
0 0 0 0 0 0 0 0 0
1 1 x x+1 x2 x2 + x x2 + 1 x2 + x + 1
x x2 x2 + x 2
x +x+1 x+1 x2 + 1 1
x+1 x2 + 1 x+1 x2 + 1 0 x2 + x
x2 1 x2 + x x2 + 1 x
2
x +x x2 + 1 0 0
x2 + 1 0 x2 + 1
2
x +x+1 x2

Since the multiplication is commutative, the table is symmetric about the


diagonal.

3. Let R be the ring Q[x]/hx3 + 2x2 x 3i. Describe the elements of R


and give the formulas necessary to describe the product of any two elements.

We see that Q[x]/hx3 + 2x2 x 3i = {rx2 + sx + t|r, s, t Q}. Using the


relations x3 = 2x2 + x + 3 and x4 = x3 x = 2x3 + x2 + 3x = 5x2 + x 6,
we can nd the product of any two elements in R. (The back of the book gives
something less useful.)

4. Give a multiplication table for the ring Z3 [x]/hx2 1i.

We see that Z3 [x]/hx2 1i = {ax + b|a, b Z3 } = {0, 1, 2, x, x + 1, x +


2, 2x, 2x + 1, 2x + 2}. It is left to the reader to write out the table. Simply use
2
the relation x = 1 to simplify products.

5. Show that Q[x]/hx2 2i


= Q[x]/hx2 + 4x + 2i.

Dene : Q[x]/hx2 2i Q[x]/hx2 + 4x + 2i via (f (x)) = f (x + 2). We


2
have seen from Q8 from 5.2 that is a homomorphism. We see that (x 2) =
x +4x+2, so (hx 2i) = hx +4x+2i. Thus, Q[x]/hx 2i
2 2 2 2 2
= Q[x]/hx +4x+2i
by Example 5.3.5.

6. Let R = F [x] and let I be any ideal of R.


(a) Prove that there is a unique monic polynomial f (x) with I = hf (x)i.
(b) Prove that if I is a maximal ideal of R, then I = hp(x)i for some monic
irreducible polynomial p(x).
CHAPTER 5. COMMUTATIVE RINGS 19

(a) We know that since F [x] is a principal ideal domain, we have that
I = hg(x)i for some g(x) F [x]. Suppose g(x) = an xn + a1 x + a0 .
1
Then, f (x) = an g(x) is a unique monic polynomial with the same roots
as g(x). Let h(x) F [x]. Then, if h(x)/g(x) = q(x) + r(x), then h(x)/f (x) =
h(x)/(a1
n g(x)) = (h(x)/g(x)) an = an q(x) + an r(x). Thus, the map :
F [x]/hg(x)i F [x]/hf (x)i dened by (h(x)) = an h(x) gives a one-to-one,
onto map which gives that F [x]/hg(x)i = F [x]/hf (x)i. Thus, I = hf (x)i.

(b) From (a), we know that if I F [x], then I = hp(x)i, where


is an ideal of
p(x) is monic. Thus, we only need to show that if I is maximal, then p(x) is
irreducible. Suppose, then, that p(x) is reducible. Then p(x) = f (x)g(x) for
some f (x) 6= g(x) F [x]. Clearly, hf (x)g(x)i hg(x)i, hf (x)i. Thus, I is not
maximal.

7. Show that the intersection of two ideals of a commutative ring is again


an ideal.

Suppose I J are ideals of the commutative ring R. Then, let x, y I J


and
and r R. Thenx y I J since x y I and x y J (by denition of
ideal). We also see that r x I J : since x I and I is an ideal, r x I ;
also, r x J since J is an ideal. Thus, I J is an ideal.

8. Show that if R is a nite ring, then every prime ideal is maximal.

Let I be a prime ideal of R. Suppose that J is an ideal of R such that


I J R. Let j J . Since R is nite, there must be n > m Z+ so that
j m = j n . Thus, j m j n = 0 I . This implies that j m (1 j nm ) I . Now,
m
since I is prime, this implies that either j I or (1 j nm ) I . If j m I ,
m1
then we have jj I , implying that j I or j m1 I . If j I , then I = J
m1
and the problem is solved. Let us suppose that j I . Thus, jj m2 I .
m2
Again, either j I , or j I . Continuing this factorization, we get that
jj m(m1) = jj I which implies j I . Thus, j I and I = J .
nm
If (1 j ) I , then j m (1 j nm ) = 0 I (remembering that j m = j n ).
m nm
Now, j (1 j ) (1 j nm ) = (j m 1)(1 j nm ) = j m j n 1 + j nm =
nm
j 1 I (by the closure property. Note that j m (1 j nm ) (1 j nm ) =
(1 j nm ). Thus, j m 1 = 1, showing that j m = 0 I . Then, by the
argument in the previous paragraph, j I . Thus, I = J . [Somehow I think that
something is wrong with this paragraph, but I can't nd any invalid statement.]

9. Find a nonzero prime ideal of Z Z that is not maximal.

Let I = pZZ where p is prime. Then, I is a prime ideal since (a, b)(c, d) I
implies that a = kp or b = kp for some k Z. Thus, I is prime. We will show
(Q25) that the only possible prime ideals are those of the form pZ Z where
p is prime. However, (Z Z)/(pZ Z) = Zp which is a eld, so the ideal
CHAPTER 5. COMMUTATIVE RINGS 20

is maximum. Thus, the only nonmaximal primes are 0Z or Z0 since


Z/(0 Z)
=Z which is not a eld.

10. Let P be a prime ideal of the commutative ring R. Prove that I and J
are ideals of R and I J P , then either I P or J P .

Supposee that there are x I, y J such that x, y


/ P . Then, since I and J
are ideals, they have the absorption property and so xy I J . But I J P
and since P is prime, this implies that xP or y P . Contradiction.

11. Let R be a commutative ring, with a R. The annihilator of a is


dened by
Ann(a) = {x R|xa = 0}.
Prove that Ann(a) is an ideal of R.

Let x, y Ann(a). Then, (x y) a = xa ya = 0 0 = 0. Thus,


x y Ann(a). Let r R and x Ann(a). Then, (r x) a = r 0 = 0. Thus,
rx Ann(a). Thus, Ann(a) is an ideal.

12. Recall that an element of a commutative ring is said to be nilpotent if


an = 0 for some positive integer n.
(a) Show that the set N of all nilpotent elements of a commutative ring forms
an ideal of the ring.
(b) Show that R/N has no nonzero nilpotent elements.
(c) Show that N P for each prime ideal P of R.

(a) Let x, y N . xm = y n = 0 for some m, n Z+ .Suppose,


Then,
 without

k mn
mn
Pmn mnk k
loss of generality, that m n. Now, (xy) = k=0 (1) x y =
k
   
Pm(n1) mn mnk k Pmn mn mnk k
k=0 (1)k x y + k=m(n1)+1 (1)k x y . We see
k k
k k
that both sums are zero since x = 0 for all k > m and y = 0 for all k > n.
Thus, x y N .
m
Next, suppose r R and x N . Then, (r x) = rm xm = r 0 = 0. Thus,
rx N . Thus, N is an ideal.

(b) Let x + N R/N be such that (x + N )m = xm + N = N for some


+ m m
mZ . Since x + N = N if and only if x = 0, this implies that R/N has
no nonzero nilpotent elements.

(c) Let x N . Then, xm = 0 for some m Z+ . Then, 0P for all prime


ideals P , so 0 = x xm1 P , implying that x P .

13. Let R be a commutative ring with ideals I, J . Let


I + J = {x R|x = a + b for some a I, b J}.
CHAPTER 5. COMMUTATIVE RINGS 21

(a) Show that I + J is an ideal.


(b) Determine nZ + mZ in the ring of integers.

(a) Let x, y I + J . Then, x = a + b and y = c + d for a, c I and b, d J .


Then, x y = (a c) + (b d). Since a c I and b d J , we have
x y I + J.
Next, let r R and x I + J where x = a + b with a I and b J . Then,
r x = r(a + b) = ra + rb. Since I and J are ideals, ra I and rb J . Thus,
rx I + J . Thus, I + J is an ideal.

(b) Suppose that gcd(n, m) = d. Then, n = dq1 and m = dq2 . Then, let
rdq1 + sdq2 nZ + mZ where r, s Z. Then, rdq1 dZ and sdq2 dZ, so
nZ + mZ dZ.
Since d = gcd(m, n), we have d = rm + sn for some r, s Z. Thus, d
nZ + mZ.
Thus, mZ + nZ = gcd(m, n)Z.

14. Let R be a commutative ring with ideals I, J . Let


( n )
X
+
IJ = ai bi |ai I, bi J, n Z .
i=1

(a) Show that IJ is an ideal contained in I J .


(b) Determine (nZ)(mZ) in the set of integers.
P P
(a) Let x, y IJ . Then, x y = ai bi aj bj IJ . Thus, x y IJ .
Since ai bi I J for all i, we also have x y I J .
P P
Next, let r R and x = ai bi IJ . Then, rx = rai bi . Since I and J
are ideals, we have rai I for all ai , so rx IJ . On the other hand, rbi J
for all bi , so rx I J as well.

PN P
(b) Suppose that x= i=1 ri nsi m where ri , si Z. Then, x = mn ri si .
Clearly x mnZ. Thus, (mZ)(nZ) mnZ.
Next, suppose x mnZ. Then x = kmn for some k Z. We could write
x = (km)(1 n) (mZ)(nZ). Thus, mnZ (mZ)(nZ).
Thus, (mZ)(nZ) = mnZ.

15. Let M = {f (x, y) F [x, y]|f (0, 0) = 0} be the maximal ideal of F [x, y]
dened in Example 5.3.8.
(a) Show that M = {s(x, y)x + t(x, y)y|s(x, y), t(x, y) F [x, y]}.
(b) Using the denition in Exercise 14, nd M .
2

(a) Since [s(x, y)x+t(x, y)y](0,0) = 0, we clearly have {s(x, y)x+t(x, y)y|s, t
F [x, y]} M .
CHAPTER 5. COMMUTATIVE RINGS 22

Next, if f (0, 0) = 0, then the constant term of f must be zero. Hence,


collect all the terms divisible by x into one set of brackets and everything else
(which must be divisible by y since the constant term is zero) into another
set. Call the rst expression s(x, y)x and the second expression t(x, y)y . Thus,
f (x, y) = s(x, y)x + t(x, y)y .

M 2 = (s1,i (x, y)x+t1,i (x, y)y)(s2,i (x, y)x+t2,i (x, y)y) = [s1,i (x, y)s2,i (x, y)x2 +
P P
(b)
(s1,i (x, y)t2,i (x, y) + t1,i (x, y)s2,i (x, y))xy + t1,i (x, y)t2,i (x, y)y 2 ]. Not that there
2
is no constant term. Thus, M = M .


16. Let R = {m+n 2|m, n Z} and let I = {m+n 2|m, n Z and m is even}.
(a) Show that I is an ideal of R.
(b) Find the well-known commutative ring to which R/I is isomorphic.


x = 2a + b 2, y = 2c + d 2 I . Then x y =2(a c) + (b d) 2.
(a) Let
Thus, x y I . Then, if x = 2a + b 2 I and r = c + d 2 R, we have rx =

(2a+b 2)(c+d 2) = 2ac+4bd+(2ad+bc) 2 = 2(ac+2bd)+(2ad+bc) 2 I .
Thus, I is an ideal.


(b) Let us nd the congruence classes.
Anything of the form 2a + b 2 is
in I, so anything of the form (2a + 1) + b 2 is not. These are clearly
the only
two classes that partition
R. Dene : R Z2 via (2a + b 2) = [0]2 and
(2a + 1 + b 2) = [1]2 . Then, this isclearly one-to-one
and onto. We only

really need to show that [(2a + 1) + b 2][(2c + 1) + d 2] = (2e + 1) + f 2,
which will be left to the reader (we have shown everything else). Then we will
see that the multiplication- and addition tables for R are the same as for Z2 .
Thus, R/I
= Z2 .
 
a b
17. Let R be the set of all matrices over Q such that a = d and
c d
c = 0.
(a) Verify that R is a commutative ring.
(b) Let I be the set of all matrices for which a = d = 0. Show that I is an
ideal of R.
(c) Use the fundamental homomorphism theorem for rings to show that
R/I = Q.

(a) (Closure of +) Given since Q is closed under +.


(Associativity) Given since + is associative in
  Q.    
0 0 a b 0 0
(Zero element) is the zero element since + =
0 0 c d 0 0
 
a b
.
c d    
a b a b
(Additive Inverses) is the additive inverse of .
c d c d
(Commutativity of +) Given since + is commutative in Q.
CHAPTER 5. COMMUTATIVE RINGS 23

       
a b c d ac ad + bc c d
(Commutativity of ) = =
  0 a 0 c 0 ac 0 c
a b
.
0 b  
1 0
(Unity element) is the unity element.
0 1
(Distributivity) Given because matrix multiplication is distributive.
 
1 0
Thus, R is a commutative ring with 1 = .
0 1
         
0 a 0 b 0 a 0 b 0 ab
(b) Let , I . Then, =
0 0 0 0 0 0 0 0 0 0
       
c d c d 0 a 0 ac
I . Now, let R. Then, = R. Thus,
0 c 0 c 0 0 0 0
I is an ideal of R.
 
a b
(c) Dene : R Q by = a. Clearly, (R) = Q. Also,
   0 a
a b
ker = R a = 0 . Clearly, ker = I . By the fundamental ho-
0 a
momorphism theorem, R/ ker = (R), so R/I = Q.

18. Let R be a commutative ring with ideals I, J such that I J R.


(a) Show that J/I is an ideal of R/I ,
(b) Show that the factor ring (R/I)/(J/I) is isomorphic to R/J .
(c) Show that J/I is a prime (or maximal) ideal of R/I if and only if J is
a prime (or maximal) ideal of R.

a + I, b + I J/I . Then, (a + I) (b + I) = (a b) + I = c + I J/I .


(a) Let
Since R/I = {x + I|x R}, we know (a + I) (b + I) R/I . Next, suppose
r R. Then, r(a + I) = ra + I R/I .

(b) Let : R/I R/J (r + I) = r + J . (This is the natural


be dened by
projection mapping.) Then, since I J R, we know that the map is onto,
so (R/I) = R/J . Also, the elements that are mapped to zero (aka J ) are the
elements of the form j + I , where j J . These are precisely the elements from
J/I . Thus, ker = J/I . Then, by the fundamental theorem of homomorphisms,
we have (R/I)/(J/I) = R/J .

(c) (Prime) () Suppose that J is a prime ideal of R and that we have


(a + I)(b + I) = ab + I J/I . Since ab + I J/I , we have ab J and we know
that a J or b J . Thus, a + I J/I or b + I J/I . Thus, J/I is a prime
ideal of R/I .
() Suppose that J is not a prime ideal of R. Then, there are x, y R such
that xy J , yet x / J and y / J . Then, xy + I = (x + I)(y + I) J/I , yet
since x, y
/ J , we know that (x + I), (y + I)
/ J/I .
CHAPTER 5. COMMUTATIVE RINGS 24

(Maximal) By the correspondence theorem, we know that there is a one-to-


one correspondence between the ideals of R/I and the ideals of R that contain
I. We showed in (a) that J/I R/I , so then this ideal corresponds
is an ideal of
to 1 (J/I) = J .
() Suppose that J is maximal and that K is an ideal of R such that
K J/I . Then, we know that 1 (J/I) = J , and since K J/I , 1 (K) J .
1
Since J is maximal, we have (K) = J , and so K = J/I .
() Suppose that J is not maximal and that K J/I is an ideal of R/I
with k K , but k / J/I . Then, 1 (k) / J and since 1 (K) J (since
1
K J/I ), we conclude that (K) ) J . Thus, J is not maximal.

19. Use Exercise 18 together with Proposition 5.3.9 to determine all prime
ideals and all maximal ideals of Zn .

By Q18, I/nZ is a prime ideal of Z/nZ if and only if I is a prime ideal of Z.


Thus, the prime ideals of Z/nZ = Zn are the ideals of the form hpi/nZ where p
is a prime number. Since in Z, prime = maximal, we have also found all of the
maximal ideals.

20. In the ring Z[i] of Gaussian integers, let hpi be the ideal generated by a
prime number. Show that Z[i]/hpi has p2 elements, and has characteristic p.

We want to show that any element in Z[i]/hpi has the form [a]p + i[b]p . If
we can show this, then clearly |Z[i]/hpi| = p2 and char(Z[i]/hpi) = p.
Now, all multiples of p are in hpi. We know that any number without an
imaginary part which is not divisible by p is of the form [a]p , which are not in
hpi, and so form separate equivalence classes. Similarly, pure imaginary numbers
form classes of the form i[b]p . Finally, combinations of two classes of these types
are also not in hpi and so form even more congruence classes. This combination
completely classies all of Z[i] into congruence classes. Thus, any number in
Z[i]/hpi has the form [a]p + i[b]p , so our conclusion follows.

21. In the ring Z[i] of Gaussian integers nd necessary and sucient condi-
tions on integers m and n for the element m + ni to belong to the ideal h1 + 2ii.
Use these conditions to determine the ideal h1 + 2ii Z of Z.

h1 + 2ii = Z[i] (1 + 2i). Thus, any element in h1 + 2ii must have


Recall that
form (a + ib)(1 + 2i) = a 2b + i(2a + b), where a, b Z.
Clearly, h1 + 2ii Z contains precisely the elements of the form a 2b
where 2a + b = 0. This implies that b = 2a, so elements must have the form
a 2(2a) = a + 4a = 5a. Thus, h1 + 2ii Z = 5Z.

22. In the ring Z[i] of Gaussian integers, show that the ideal h5 ii is not
a prime ideal.
CHAPTER 5. COMMUTATIVE RINGS 25

We see that5i = (1i)(3+2i). We need to check that 1i, 3+2i / h5ii.


We see that h5 ii has elements of the form (a + ib)(5 i) = (5a + b) + i(5b a).
If 1 i h5 ii, we would have to have a, b Z such that 5a + b = 1 and
3 2
5b a = 1. This implies that a = 13 and b =
13 . Since neither is in Z, we
conclude that 1 i / h5 ii.
Now, if 3 + 2i h5 ii, we would have to have a, b Z such that 5a + b = 3
1
and 5b a = 2. This implies that a = b =
2 . Again, we conclude that
3 + 2i
/ h5 ii.
Now, we have found that (1 i)(3 + 2i) = (5 i) h5 ii, but 1 i, 3 + 2i /
h5 ii. Thus, h5 ii is not a prime ideal.

23. Let R be the set of all continuous functions from the set of real numbers
into itself. In Exercise 13 of Section 5.1, we have shown that R is a commutative
ring if the following formulas
(f + g)(x) = f (x) + g(x) and (f g)(x) = f (x)g(x)

for all x, are used to dene addition and multiplication of functions. Let a be
a xed real number, and let I be the set of all functions f (x) R such that
f (a) = 0. Show that I is a maximal ideal of R.

(I is an ideal) Letf (x), g(x) I . Then, (f g)(x) = f (x) g(x). We


see (f g)(0) = 0 0 = 0. Thus, (f g)(x) I . Let h(x) R. Then,
(h f )(x) = h(x)f (x). We see that (h f )(0) = h(0) 0 = 0. Thus, (h f )(x) I .
(I is maximal) We see that I is the set of polynomials with constant term
equal to 0. Thus, if we take the factor ring R/I we obtain the polynomials over
R which only have constant terms. To see this, note that we obtain an element
of R by taking an element of I and adding a real number. Thus, any element
of R is of the form a + I , where a R. This ring is isomorphic to R (by the
natural mapping (a + I) = a), which is a eld. Thus, since R/I = R, by
Proposition 5.3.9, I is maximal.

24. Let I be the smallest ideal of Z[x] that contains both 2 and x. Show that
I is not a principal ideal.

The smallest ideal that contains 2 and x is h2i hxi. (We know that hai
is the smallest ideal that contains a. We have also shown that the intersection
of two ideals is an ideal, and we know that the intersection will give us the
fewest number of elements.) Now, h2i = {f (x) Z[x]|all coecients are even}
and hxi = {f (x) Z[x]|f (x) has no constant term}. Thus, h2i hxi = {f (x)
Z[x]|f (x) has even coecients with no constant term}. Now, 2 h2i hxi, so
if h2i hxi is a principal ideal, we must have f (x) Z[x] such that 2 hf (x)i.
This would imply that deg f (x) = 0. But then, such an f (x) would be unable
to generate x h2i hxi. The reason this fails is because Z does not have
multiplicative inverses, so ax b 6= x for any a, b Z.
CHAPTER 5. COMMUTATIVE RINGS 26

25. Let R and S be commutative rings, let I be an ideal of R, and let J be


an ideal of S .
(a) Show that I J as an ideal of R S .
(b) Show that (R S)/(I J) = (R/I) (S/J).
(c) Show that I J is a prime ideal of R S if and only if either I = R and
J is a prime ideal of S , or else I is a prime ideal of R and J = S .
(d) Show that if K is any ideal of R S , then there exists an ideal I of R
and an ideal J of S such that K = I J .

(a) Let (a, b), (c, d) I J . Then, (a, b) (c, d) = (a c, b d) I J , since


I and J are ideals. Next, let (r, s) R S . Then, (r, s)(a, b) = (ra, sb) I J
since I, J are ideals. Thus, I J is an ideal of R S .

(b) Dene : R S (R/I) (S/J) via (r, s) = (r + I, s + J). Clearly,


(R S) = (R/I) (S/J). The elements that are mapped to 0 (aka (I, J))
are any element of the form (i, j) where i I and j J . Thus, ker = I J .
By the fundamental theorem for homomorphisms, we have (R S)/(I J) =
(R/I) (S/J).

(c) By Proposition 5.3.9, I J is a prime ideal of R S if and only if


(R S)/(I J) is an integral domain. By (b), (R S)/(I J) = (R/I) (S/J).
Thus, I J is prime if and only if (R/I) (S/J) is an integral domain. Now, for
any I, J , we can always form the product (i, 1)(1, j) = (i, j) where i I , j J .
This product will always be such that (i, 1)(1, j) I J , but (i, 1), (1, j)
/ I J
unless 1 I (in which case I = R) or 1 J (in which case J = S ).
Without loss of generality, suppose J = S . Then, (R S)/(I S) = R/I
S/S = R/I . Thus, (RS)/(I J) is an integral domain if and only if J = S and
R/I is an integral domain. This happens if and only if I is prime (Proposition
5.3.9) and J = S .

(d) Suppose that K is an ideal of R S . Then, if (a, b), (c, d) K , and


(r, s) R S , we must have (a c, b d) K and (ra, sb) K . This implies
that the coordinates are closed under addition and enjoy the absorption property
separately. Thus, K =I J for ideals IER and J E S.

26. Let R be the set of all rational numbers m/n such that n is odd.
(a) Show that R is a subring of Q.
(b) Let 2 R = {m/n R|m is a multiple of 2 and n is odd}, for any posi-
k k

tive integer k. Show that 2 R is an ideal of R.


k

(c) Show that each proper nonzero ideal of R has the form 2 R, for some
k

positive integer k.
(d) Show that R/2 R is isomorphic to Z2k .
k

(e) Show that 2R is the unique maximal ideal of R.


CHAPTER 5. COMMUTATIVE RINGS 27

a(2n+1)+b(2m+1)
(a) Let
a b a b
2m+1 , 2n+1 R. Then, 2m+1 + 2n+1 = (2m+1)(2n+1) = a(2n+1)+b(2m+1)
4mn+2(m+n)+1
a a
R. If 2m+1 R, then clearly 2m+1 R. Thus, R is a subring of Q.

m2k `2k m2k `2k m2k (2j+1)+`2k (2n+1)


(b) Let
2n+1 , 2j+1 I . Then,
2n+1 + 2j+1 = (2n+1)(2j+1) =
2mj2k +m2k +2`n2k +`2k (2mj+m+2`n+`)2k a a
(2n+1)(2j+1) = (2n+1)(2j+1) 2k R. If 2b+1 R, then 2b+1
k k
m2 am2 k k
2n+1 = 4bn+2(b+n)+1 2 R. Thus, 2 R is an ideal of R.

m
(c) Suppose I is an ideal of R d where m and
that contains the element
d are odd. Then, by the absorption property, we should be able to obtain any
a
element of R. For instance, if we wish to obtain the element
b R where b is
ad m a
odd, we have the product
mb d = b . Thus, I = R.

(d) We see that 2k R is a principal ideal (as its notation suggests). Now,
let us determine the cosets of h2k i.
r R h2k i, then r = s2k , where
If
s h2 i. Otherwise, by the division algorithm, r = qs2k + r0 , where 1 r < 2k .
k
0
Based on the value for r , we nd the coset for any r R. Use the mapping
(r + 2 R) = [r]2k to see that R/2k R
k
= Z2k .

(e) We have found that the only ideals of R are 2k R for some k . Clearly,
k k
2 m2k1
2R 4R 2 R . (For instance m2
k
n 2R since m2n = 1 n .)
Thus, 2R is the unique maximal ideal.

5.4 Quotient Fields

Notational convention:
a
The notation
b is equivalent to [a, b] . They
both refer to the equivalence relation that builds Q(D) for an integral domain
D.
1. Complete the proof of Lemma 5.4.3, to show that multiplication of equiv-
alence classes in Q(D) is well-dened.
p1 p2 pk m
a
= qq11 qq22 q `r
m c
r a c ac
Let
b = p1 p2 pk n n , d s . Then, = =
(p1 pk )(q1 qn )mr  mr  m  r  q` s b d bd

(p1 pk )(q1 qn )ns ns = n s .

2. Show that the associative and commutative laws hold for addition in
Q(D).
  h i h i h i
a c e
a cf +ed adf +b(cf +ed)
(Associative)
b
+ d
+ f = b
+ df = bdf .

(1)
h i h i h i
a c  e
 ad+bc  e f (ad+bc)+ebd
b + d + f = bd
+ f = bdf . (2)

We see that (1) = (2), so + is associative.
a c  ad+bc   bc+ad  c a
(Commutative)
b + d = bd
= db
= d + b . Thus,
+ is commutative.
CHAPTER 5. COMMUTATIVE RINGS 28

3. Show that the associative and commutative laws hold in Q(D).


  h i 
a h i h i
c e
a ce ace
= ac
 
(Associative)
b d f = b df = bdf bd
h i h i
e
= ab dc fe .
    
f
 a   c  ac   ca  c a
(Commutative)
b d = bd = db = d b .

4. Let : D Q(D) be the mapping (D) = [d, 1] dened in Theorem


5.4.4. Show that is an isomorphism if and only if D is a eld.

() If is an isomorphism, then every element in Q(D) can be written as


h 0i
d
Q(D) is a eld, so for any 1 Q(D), we must have d1
d
1 . Now,

 d  h d0 i h dd0 i  1  d
Q(D) such that 1 1 = 1 = 1 . Thus, for any 1 Q(D), we have
  h 0 i h 0i
1 = 1 11 = 1 d1 d1 = 1 d1 1 d1 = d d0 . Thus, any d D
   

has an inverse, so D is a eld.


() If D is a eld, then D contains all inverses.
(1-1) is a direct map and is thus one-to-one.
h i
x
(Onto) Let
y D . Since the numerators and denominators come from D ,
h i   h 1 i
a eld, this expression is equivalent to
x
= x1 y 1 = (xy 1 ).
 xy   x  yy 
(Preservation) (xy) = = 1 = (x)(y).
 x 1y 1
(x + y) = x+y

1 = 1 + 1 = (x) + (y).

5. In Theorem 5.4.6, verify that is a one-to-one ring homomorphism.

= dc . This means that (a)(b)1 = (c)(d)1 .


a  
(1-1) Suppose
b
1
Now, F is a eld, so the previous equality is the same as (c) (b)1 =
(a)1(d) 1
1
(cb) = 1
(ad). Since is one-to-one, this implies cb = ad.
a
  c   adbc   0  a c
Thus,
b d = bd  = bd = [0] . Thus,
 ad+bc  b
= d .
(Preservation of +) a c = (ad + bc)(bd)1 = ((ad) +
b + d = bd
1 1 1 1 1
(bc))(bd)
  = ((a)(d) + (b)(c)) (b) (d) = (a)(b) + (c)(d)
= ab + dc .

a c = ac = (ac)(bd)1 = (a)(c)(b)1 (d)1 =
   
(Preservation of )
b  d   bd
(a)(b)1 (c)(d)1 = a c . b d

6. Let D1 and D2 be integral domains, with quotient elds Q(D1 ) and


Q(D2 ), respectively. Let : D1 D2 be a ring homomorphism.
(a) Prove that if is one-to-one, then there exists a ring homomorphism
: Q(D1 ) Q(D2 ) such that ([d,
1]) = [(d), 1] for all d D1 .
(b) Prove that if is not one-to-one, then it is impossible to nd a ring
homomorphism : Q(D1 ) Q(D2 ) that satises the conditions of part (a).
  h i  
1 = (1) = 1 since is a homomorphism.
(a) (Identity)
1 1 1
CHAPTER 5. COMMUTATIVE RINGS 29

h i h i
(a+c) (a)+(c)
=
a  c   a+c 
(Preservation of +)
1 + 1 1 = 1 = 1 =
h i h i
(a) (c)
+ (c) .
1 +
= (a) 1
h i h i h i
a b ab = (ab) = (a) (b) =
     
(Preservation of ) =
1 1 1 1 1 1
(b)
(a) .

(b) If
h one,i then hthereiare x 6= y D1 such that (x) = (y).
is not one-to
= (x+y) = 2 (x)
x y
Then, , but since x 6= y , x + y 6= 2x and
1 + 1
 x+y  h (x+y) i h1 i 1
(x)
1 = 1 6= 2 1 .

7. Determine Q(D) for D = {m + n 2|m, n Z}.
h i
a+b 2
For every a + b 2 D, we must introduce the elements
1 and
h i h i h i
1 1 1
a+b 2 h
. For two elements
a+b 2
+ c+d 2
, we have the equivalent ex-
i
ab 2
h i
cd 2

pression 2
a 2b2 + c 2d2
2 Q[ 2]. Thus, r + s Q[ 2] r, s Q(D).
for all

Also, doing the same sort of trickery, we easily see that
rs Q[ 2] for all
rs Q(D). Thus, Q(D) = Q[ 2].

8. Let p be a prime number and let D = {m/n|m, n Z and p - n}. Verify


that D is an integral domain and nd Q(D).
m n mn
(D is an integral domain) Let
q1 p+r1 q2 p+r2 = (q1 p+r1 )(q2 p+r2 ) = 0. The
m n
only way this can happen is if one of m or n is 0, in which case or
q p+r 1 1 q2 p+r2
would be 0. Thus, D is an integral domain.

p` p1 1 pkk
For the element
p1 1 p
pi 6= p for all i, the element p1` must
n , where
n
be introduced to Q(D). This means that Q(D) must have elements whose
denominator can contain any power of p. Since D already has elements that
can contain other primes, we simply have Q(D) = Q.

9. Determine Q(D) for D = {m + ni|n, m Z} C.

Very similar to Q7, Q(D) = Q[i].

10. Considering Z[x] as a subring of Q[x], show that both rings have the
same quotient elds.
1 k
Since nxk Z[x],
PQ(Z[x])nx
surelyfor all n, k Z. Then, since
m k
Q(Z[x]) x Q(Z[x])
is a eld, this implies that
n for all m, n, k Z. We
now see that Q[x] Q(Z[x]). Since Q(Z[x]) is a eld, it must contain all of the
inverses for Q[x]. Thus, Q(Q[x]) Q(Z[x]). On the other hand, we have that
Z[x] Q[x], so by the same argument about Q(Q[x]) being a eld, we have
Q(Z[x]) Q(Q[x]). Thus, Q(Z[x]) = Q(Q[x]).
CHAPTER 5. COMMUTATIVE RINGS 30

11. Show that if P is a prime ideal of D, then DP = {a/b Q(D)|b / P }


is an integral domain with D DP Q(D).
 a   c   ac 
(DP is an integral domain) Suppose that
b d = bd = [0]. Now, since
P is an ideal, 0 P , so neither b nor d can equal 0. Also, since D is an integral
domain, we cannot have bd = 0 since b, d 6= 0. Thus, ac = 0 and since D is an
a
integral domain, either a = 0 (in which case
c b = [0]) or c = 0 (in which case
d = [0]). Thus, DP is an integral domain.
Clearly, DP Q(D), since the elements come from Q(D).
Let a D . Then, by denition of DP , a DP . Thus, D DP . (It might
be worth pointing out that 1 / P since P is an ideal.)

12. In the ring DP dened in Exercise 11, let M = {a/b DP |a P }.


(a) Show that M is an ideal of DP .
(b) Show that DP /M = Q(R/P ), and conclude that M is a maximal ideal
of DP .
h i h i h h i i h i
(a) Let
p1
q1 , pq22 M . Then, pq22 = p1 qq21p
p1
q1 q2
2 q1
. Since P is
h i h i h i
p1 p2
an ideal, p1 q2 p2 q1 P , and so
q1 q2 M . If qd0 DP , then
h i h i h i h i h i
d p dp d p
q 0 q = q 0 q . Since P is an ideal, dp P , so
q 0 q M . Thus, M is
an ideal of DP .
(b) I can only assume that there was a misprint in this question as R is not
dened. My best guess is that R shouldhbe Di.
ab = a+P

If that is the case, dene
b+P . We need to show that is a
i h
a c a c ad+bc ad+bc+P
 
homomorphism. First, if
b, d DP , then + = = b d bd bd+P .

a
 c
 h a+P i h c+P i h (d+P )(a+P )+(b+P )(c+P ) i h ad+bc+P i
Also, +
b d = b+P + d+P = (b+P )(d+P ) = bd+P .

Thus, + is preserved.
h i h i h i
a c ac ac+P a+P c+P a c
   
Next, b =
d bd = bd+P = b+P d+P = b + d . Thus,
is preserved. Thus, is a homomorphism.
We see that the elements that get mapped to 0 (which is to say that the
numerator of the function value is P ) are the ones whose numerators are mem-
bers of P. This set is precisely M . Thus, by the fundamental theorem for
homomorphisms, we see that DP /M = Q(D/P ).

13. Let R be a commutative ring. A derivation


on R is a function : R
R such that (i) (x+y) = (x)+(y) and (ii) (xy) = (x)y+x(y). Show that
if is a derivation on an integral domain D with quotient eld Q(D), then can
be extended to a derivation of Q(D) by dening (a/b) = (b(a) a(b))/b2
for all a, b D with b 6= 0.
CHAPTER 5. COMMUTATIVE RINGS 31

 bd(ad+bc)(ad+bc)(bd)
a c ad+bc
= bd[(ad)+(bc)](ad+bc)[d(b)+b(d)]

(i)
b + d =
bd = b2 d2 b2 d2 =
[bda(d)+bd (a)+b d(c)+bdc(b)][ad (b)+adb(d)+bcd(b)+b2 c(d)]
2 2 2
bd2 (a)+b2 d(c)ad2 (b)b2 c(d)
b2 d2 = b2 d2 =
d2 [b(a)a(b)]+b2 [d(c)c(d)]
. (1)
b2 d2
 b(a)a(b) d(c)c(d) 2 2
a c
= d [b(a)a(b)]+b [d(c)c(d)]

Then,
b + d = b2 + d2 b2 d2
(2). Now, (1) = (2), so (i) is satided.
 bd(ac)ac(bd)
a c ac
= bd[a(c)+c(a)]ac[b(d)+d(b)]

(ii)
b d = bd =  b2 d2 b2 d2 . (1)
a c a c b(a)a(b) c a d(c)c(d) c[b(a)a(b)]
Next,
b d + b d = b2 d+b d2 = b2 d +
a[d(c)c(d)] cd[b(a)a(b)]+ab[d(c)c(d)] bd[a(c)+c(a)]ac[b(d)+d(b)
bd2 = b2 d2 = b2 d2 . (2)

Now, (1) = (2), so (ii) is satised. Thus, is a derivation on Q(D).

14. Show that : Q[x] Q[x] dened by (f (x)) = f 0 (x) for all f (x)
Q[x] is a derivation. Describe the derivation dened on the quotient eld of
Q[x].

We already know that (f (x) + g(x)) = f 0 (x) + g 0 (x) and (f (x)g(x)) =


0 0
g(x)f (x) + f (x)g (x) from calculus. Thus, is a derivation of Q[x]. We have
shown that Q(Q[x]) = Q(Z[x]), which is just the set of rational functions.
 
f (x)
Thus, g(x) should be dened according to the ordinary quotient rule. That

g(x)f 0 (x)f (x)g 0 (x)


 
f (x) g(x)(f (x))f (x)(g(x))
is, g(x) = (g(x))2 = (g(x))2 .
Chapter 6

Fields
6.1 Algebraic Elements

1. Show
that the following complex numbers are algebraic over Q.
(a)
2 +
(b)
nZ
n, for
(c)
p3 + 5
(d) 2 + 3
(e) (1 +

3
3i)/2
(f ) 2+ 2

(a) x2 2 has 2 as a root.
2
(b) x n has
n as a root.

(c) Let x =
3 + 5. Then, x 3 = 5. Squaring both sides, we get
x2 2 3x + 3 = 5 which is the same thing as x2 2 = 2 3x. Again, squaring
4 2 2 4 2
both sides, we get x 4x + 4 = 12x . Thus, the polynomial x 16x + 4 has

3 + 5 as a root.p
(d) Let x = 2 + 3. Then, x2 = 2p+ 3, so x2 2 = 3. Squaring,

x4 4x2 + 4 = 3. Thus, x4 4x2 + 1 has 2 + 3 as a root.
i( 2
3 ) . Thus, if x = ei( 3 ) , then x3 = e2i = 1.
2
(e) Note that (1+ 3i)/2 = e
3
Thus, x 1 has the number as a root.

(f ) Let x =

3
2+ 2 = 21/3 + 21/2 . Then,x 21/2 1/3
= 2 3 . Cubing,
3 2 3 2
3 2 2x + 6x 2 2 = 2. Thus, x + 6x 2 = 3 2x + 2 2, so x + 6x 2 =
x
2(3x + 2).
6 4 3 2 4 2
Squaring, we get x + 12x 4x + 36x 24x + 4 = 2(9x + 12x + 4).
6 4 3 2
Thus, x 6x 4x + 12x 24x 4 has the number as a root.

2. Let F be an extension eld of K , and let u be a nonzero element of F


that is algebraic over K . Show that u1 is also algebraic over K .

32
CHAPTER 6. FIELDS 33

Let f (x) = an xn + +a1 x+a0 K[x] be such that f (u) = 0. Then, examine
g(x) = a0 xn + +an1 +a n . (The coecients are in the reverse
Pnorder.) We will
1
P n k n k
show that g(u ) = 0P
. If k=0 u ank = 0 , then surely u k=0 u ank =
n
0. This is the same as k=0 unk ank which Pn we are given to be equal to 0. Thus,
k
it must be true that our rst statement,
Pn k=0 u ank = 0,Pis true. (Note that
n
K[x] is an integral domain and so k=0 u ank = 0 un k=0 uk ank = 0
k

since u 6= 0.)

3. Suppose that u is algebraic over the eld K , and that a K . Show that
u + a is algebraic over K , nd its minimal polynomial over K , and show that
the degree of u + a over K is equal to the degree of u over K .

It is pretty clear that if u F , we must have K(u) F . So, since u + a =


u + a 1 K(u) (keeping in mind the notion of K(u) being a vector space
2
over K with basis {1, u, u , ...}), we see that u + a F . Furthermore, since
K(u) = K(u + a), the degrees of the extension are also equal. However, since we
have to nd the minimal polynomial anyway, we don't really need the preceeding
argument.
Note that f (x + a) gives the graph of f (x) shifted left a units. Thus, if
f (x) has a root at u, then f (x + a) has a root at u a. We need to prove this,
though, since there is no notion of shifting left in the abstract setting. Suppose
f (x) = an xn + + a1 x + a0 . Then, set f (x + a) = g(x) = an (x + a)n + +
a1 (x+a)+a0 . (This polynomial is still in K[x] since K[x] is a ring.) We see that
g(u a) = an (u a + a)n + + a1 (u a + a) + a0 = an un + + a1 u + a0 = 0.
This proves that g(x) is the minimal polynomial, for if there were a polynomial
h(x) of smaller degree for which h(u + a) = 0, we would see that h(x a) has
u as a root. Since f (x) was the minimal polynomial of u, we know that this is
impossible. Thus, g(x) is the minimal polynomial and deg f = deg g . As such,
[K(u) : K] = [K(u + a) : K].

4. Show that / Q( 2).
3

Q( space over Q has a basis {1,
We see that
2) as a vector
2}. Now,
suppose 3 Q( 2). Then 3 = ab + dc 2 for a, b, c, d Z. This means
2
d( 3a/b) a2 d2
2adb 3 +3d2
b2
that = 2. Squaring both sides, we get = 2. Since
c c2
3/ Q, we know that this is impossible. Thus, 3 / Q( 2).

5. Show that f (x) = x3 + 3x + 3 is irreducible over Q.


(a)
(b) Let u be a root of f (x). Express u and (1+u)1 in the form a+bu+cu2
1

where a, b, c Q.

(a) By the rational root theorem, the only possible rational roots are 3 and
1. By plugging these numbers in, we quickly see that f (x) is irreducible over
Q.
CHAPTER 6. FIELDS 34

(b) InQ(u), we have u(a+bu+cu2 ) = 1. This gives au+bu2 +cu3 = 1. Since


u + 3u + 3 = 0, we have u3 = 3(u + 1) and we obtain au + bu2 3cu 3c = 1
3
2 1
or (a 3c)u + bu 3c = 1. Thus, 3c = 1 c = . Then, a 3c = 0
3
a 3 3 = 0 a + 1 = 0 a = 1. Finally, b = 0. Thus, u1 = 1 31 u2 .
1
2 3 2
In Q(u), we have (1+u)(a+bu+cu ) = 1. Thus, cu +(b+c)u +(a+b)u+a =
2
1. Exploiting the same relation as above, this gives (b + c)u + (a + b 3c)u +
a 3c = 1. This implies that a = 4, b = 1, c = 1. Thus, (1 + u)1 = 4 u + u2 .

6. Show that the intersection of any collections of subelds is again a sub-


eld.
T
Let E be a eld and {Fi } x, y Fi . This
be a collection of subeld. Let
means that x and y are in every Fi . Thus, x y Fi for all i. Next, if x Fi
1
T
for all i, then since Fi are elds, we must have x Fi for all i. Thus, Fi is
a subeld of E .

7. Let F = K(u), where u is transcendental over the eld K . If E is a eld


such that K E F , then show that u is algebraic over E .

To restate what is given, we have K $ E K(u). Now, all elements in


F = K(u) are of the form a0 + a1 u + a2 u2 + where ai K . The elements
n
of K must not have u's. Since E 6= K , we must have b1 + b2 u + + bn u E
n
where bi K . But, K(b1 + b2 u + + bn u ) = K(u), so E = F , and obviously
u would have to be algebraic over E .

8. Let F be an extension eld of K .


(a) Show that F is a vector space over K .
(b) Let u F . Show that u is algebraic over K if and only if the subspace
spanned by {1, u, u2 , ...} is a eld.

(a) Since F is a eld, it is an abelian group under addition. Thus, i-v (the
axioms for vector spaces are listed on page 458) are satised.
Since K F, we know that kf F for all kK and f F. Thus, (vi)
is satised.
SinceK F and F is a eld (and thus is associative), we have k1 (k2 f ) =
(k1 k2 )f k1 , k2 K and f F . Thus, (vii) is satised.
for all
Since K F and F is a eld (and thus + distributes over ), we have
(k1 + k2 )f = k1 f + k2 f for k1 , k2 K and f F . Thus, (viii) is satised.
For the same reasons, (ix) is satised.
Since F is a eld, we must have 1 f = f for all f F. Thus, (x) is satised.
Thus, F is a vector space over K .

n
(b) () Suppose that u Then, let f (x) = x + +
is algebraic over K.
a1 x + a0 be the minimal polynomial of K[x]/hf (x)i
u. Then, = K(u) is a eld.
n
We know that K(u) is a vector space over K . It is clear that {1, u, ..., u } is a
n
basis for K(u). Thus, span{1, u, ..., u } = K(u).
CHAPTER 6. FIELDS 35

() Suppose that span{1, u, u2 , ...} is a eld. There are two possibilities:


2
(1) {1, u, u , ...} is linearly independent or (2) {1, u, u2 , ...} is not linearly inde-
pendent.
2
In the case of (1), since span{1, u, u , ...} is a eld, we must have elements
a0 + a1 u + + an u , b0 + b1 u + + bm um span{1, u, u2 , ...} such that
n

(a0 + a1 u + + an un )(b0 + b1 u + + bm um ) = c0 + c1 u + + c` u` = 1. This


would involve coecients of u terms multiplying to 0. However, the coecients
come from K which is a eld, so multiplication of nonzero elements should never
equal 0. Contradiction. Thus, (1) is not possible.
In the case of (2), then we know that there is k0 + k1 u + + k` u` = 0 where
ki 6= 0 for some i > 0. Thus, the polynomial k` x` + + k1 x + k0 has u as a
root. Thus, u is algebraic over K.

9. Let F be an extension eld of K . If u F is transcendental over K ,


then show that every element of K(u) that is not in K is also transcendental
over K .

Let u = k0 + k1 u + + kn un K(u) where ki 6= 0 for some i 1 (which


is to say the element is from K(u) K ). Suppose that u is algebraic over K .
Then, there is some polynomial over K for which u is a root. But, if we take
powers of u, we obtain an expression that has only powers of u. Thus, if we
have a polynomial that has u as a root, there must be a polynomial that has u
as a root.

10. Let u and r be positive real numbers, with u 6= 1. It follows from a


famous theorem of Gelfand and Schneider that if r is irrational and both u and
r are algebraic over Q, then ur must be transcendental over Q. You may use
this result to show that the following numbers are transcendental over Q.

3 5
(a) 7

3 5
(b) 7 +7

5
3
 5
(a) We see that 7 = 37 . Both
3
7 and 5 are both well-known

3 5
irrational numbers, so 7 is transcendental

over Q.

3 5
(b) We already proved that 7 was transcendental over Q, so by Q9
 
3 5
3 5
7 +7Q 7 Q is also transcendental.

11. Show that there exist irrational numbers a, b R such that ab is rational.

2 2
We see that (2 ) = 22 = 4 Q. (Note that 2 2
is transcendental by
Q10 and is thus irrational.)

12. Assuming that is transcendental over Q, prove that either + e or


e is irrational.
CHAPTER 6. FIELDS 36

Suppose that e = ab for a, b Z. Then, ab Q, so a minimal polynomial


c
for e is p(x) = x ( e). Thus, we have e Q()(e) = Q(, e) and
d
e Q. Thus, we have e Q, but e / Q(). This is imposssible since
Q Q(). Thus, e is irrational.

6.2 Finite and Algebraic Extensions

Notational convention: If E is an extension eld of F , we say  E/F is a(n


extesion) eld. Similarly, if V is a vector space over the eld F , we say  V /F
is a vector space. This saves lots of unnecessary typing.
1. Find the degree and a basis for each of the given extensions.
(a) Q( 3) over Q

(b) Q( 3,
7) over Q

(c) Q( 3 +
7) over Q
(d) Q( 2,

3
2) over Q

(e) Q( 2 +
3
2) over Q
(f ) Q() over Q, where = (1 + 3i)/2

(a) Basis: {1, 3} . Degree: 2.

(b) Basis: {1,
7, 21}. Degree: 4.
3,
(c) Basis: {1, 3, 7, 21}. Degree: 4.
1/2 1/3 2/3 5/6 1/6
(d) Basis: {1, 2 , 2 , 2 , 2 , 2 }. Degree: 6.
1/2 1/3 2/3 5/6 1/6
(e) Basis: {1, 2 , 2 , 2 , 2 , 2 }. Degree: 6.
i 2 i 2
(f ) Basis: {1, e 3 , e 3 }. Degree: 3.

2. Find thedegree and abasis for each of the given eld extensions.
(a) Q( 3,
over Q( 7)
21)
7) over Q(
(b) Q( 3 +
7)
(c) Q( 3, 7) over Q( 3 + 7)

(a) Basis: {1, 3}. Degree: 2.
(b) Basis: {1, 3}. Degree: 2.
(c) Basis: {1}. Degree: 1.

3. Find the degree of Q( 3 2, 4 5) over Q.

[Q( 3 2, 4 5) : Q] = [Q( 3 2) : Q][Q( 3 2, 4 5) : Q( 3 2)] = 3 4 = 12.

4. Let F be a nite extension of K such that [F : K] = p, a prime number.


If u F but u / K , show that F = K(u).

We have that p = [F : K] = [F : K(u)][K(u) : K]. Thus, [F : K(u)] = p or


[K(u) : K] = p. Since u
/ K , we must have [K(u) : K] = p, so [F : K(u)] = 1.
Thus, F = K(u).
CHAPTER 6. FIELDS 37

5. Let f (x) be an irreducible polynomial in K[x]. Show that if F is an


extension eld of K such that deg (f (x)) is relatively prime to [F : K], then
f (x) is irreducible in F [x].

Let u f (x). Then, deg (f (x)) = q[K : K(u)]. If f (x) is re-


be a root of
ducible over F [x], then u F , so we have to have K(u) F and [F : K] =
[F : K(u)][K(u) : K]. However, since gcd (deg (f (x)) , [F : K]) = 1, this is
impossible. Thus, f (x) is not reducible over F [x].

6. Let K E F be elds. Prove that if F is algebraic over K , then F is


algebraic over E and E is algebraic over K .

If F/K is algebraic, then clearly, E/K is algebraic since E F. Now,


suppose that F/K is transcendental. In particular, suppose that there is uF
which is transcendental overE . Then, by Q9 of 6.1, we have that every element
of E(u)E is transcendental over E . Let u0 E(u)E . Then, since K E , we
0
have u transcendental over K . But, E(u)E F , so we would have an element
of F being transcendental over K . But F/K is algebraic. Contradiction.

7. Let F K be elds, and let R be a ring such that F R K . If F is


an algebraic extension of K , show that R is a eld. What happens if we do not
assume that F is algebraic over K ?

Let r R K . Since R F and F/K is algebraic, there is an irreducible


p(x) K[x] such that p(r) = 0. Then, since p(x) is irreducible, we obtain
K[x]/hp(x)i = K(r) is a eld. Now, K(r)/K is a vector space, so any element
n
of K(r) can be written as k0 + k1 r + + kn r where ki K . This expression
is an element of R by the axioms for subring. (Remember that k K k R
S
since K R.) Let E = rRK K(r). We have shown (Q6 from 6.1) that
E is a subeld of F . We see that the addition and multiplication of these two
elements of E is again in R. Thus, E R. Obviously, we also have R E
(since we take K R and adjoin any missing elements from R). Thus, R = E
and is thus a eld.
If we do not have F/K algebraic, we cannot construct E, so we cannot
conclude that R is a eld.


8. Determine [Q( n) : Q] for all n Z+ .

If n is not a square, then a basis for [Q( n) : Q] is {1, n}. Thus, the
degree would be 2. If n = m2 , then, n Q, so the degree would be 1.

9. For any positive integers a, b, show that Q( a + b) = Q( a, b).

We know that a basis for
Q( a, b) is{1,
a, b, ab}. Let us now nd
a basis for Q( a + b). Any element of Q( a + b) will have the form n0 +
CHAPTER 6. FIELDS 38

n k nk
 
Pn
n1 ( a+ b)+ +nk ( a+ b)k . Examine ( a+ b)n = k=0 a b .
k

We see that every element in Q( a + b) can have a, b,
aterm involving

ab, or none of them at all. Thus, a basis for Q( a + b) is {1, a, b, ab}.

Thus, Q( a, b) = Q( a + b).

10. Let F be an extension eld of K . Let a F be algebraic over K , and


let t F be transcendental over K . Show that a + t is transcendental over K .

Suppose that (a + t) is algebraic over K . Then, there is p(x) K[x] such


that p(a + t) = 0. Thus, there is q(x) = p(x + a) K(a)[x] such that t is a root
of q(x). Thus, t is algebraic over K(a). Thus, K(a, t) = K(a)(t) = K(a). Thus
t is algebraic over K by Proposition 6.2.4. Contradiction.

11. Let F be an algebraic extension of K , and let S be a subset of F such


that S K , S is a vector space over K , and sn S for all s S and all
positive integers n. Prove that if char(K) 6= 2, then S is a subeld of F .

Let s1 , s2 S . Then, s1 + s2 S by vector addition. Since sn S for all


s S and n Z+ , we know that (s1 + s2 )2 = s21 + s22 + 2s1 s2 S . Again, by
2 2 2 2
vector addition, we have s1 + s2 + 2s1 s2 s1 s2 = 2s1 s2 S . Thus, s1 s2 S
since char(K) 6= 2. Now, distributivity in S follows from the fact that products
are closed and by the fact that S inherits the distributivity of F . Thus, S is a
ring. Since F/K is algebraic and K S F , S is a eld by Q7.

6.3 Geometric Constructions

1. Show that the roots of the polynomial 8x3 6x 1 used in Theorem 6.3.9
are u1 = cos 9 , u2 = cos 5
9 , and u3 = cos 9 .
7

3
If we make the substitution x = cos , then we obtain the expression 8 cos
6 cos 1 = 2(4 cos3 3 cos ) 1 = 2 cos(3) 1. Then, the polynomial has
(6k+1) (6k1)
the roots where 2 cos(3) = 1. This happens at = or = .
9 9
Pick any three values of to nd that cos is equal to u1 , u2 , or u3 .

2. Use the identity 4 cos3 3 cos cos(3) = 0 to show that the roots of
the polynomial x3 3x + 1 are u1 = 2 cos 2 9 , u2 = 2 cos 9 , u3 = 2 cos 9 .
4 8

Substitute x = 2 cos . The polynomial becomes (28(1cos2 )) cos +1 =


2 cos 8 cos (1 cos2 ) + 1 = 6 cos + 8 cos3 + 1. This equals 0 when
8 cos3 6 cos = 1 4 cos3 + 3 cos = 12 cos(3) = 12 . This
2(3k+1) 2(3k1)
happens at = or = . Hence, the polynomial is equal to zero
9 9
for x values u1 , u2 , and u3 .
CHAPTER 6. FIELDS 39

3. In this exercise we outline how to construct a regular pentagon. Let


= cos(2/5) + i sin(2/5).
(a) Show that is a primitive fth root of unity.
(b) Show that ( + ) + ( + 1 ) 1 = 0.
1 2

(c) Show that + = (1 + 5)/2.


1
p 
(d) Show that cos(2/5) = (1+ 5)/4 and that sin(2/5) = 10 + 2 5 /4.
(e) Conclude that a regular pentagon is constructible.

(a) By example A.5.3, this is a solution to z 5 = 1. Thus, is a primitive


fth root of unity.
(b) Plugging = cos(2)+i sin(2) where = 2/5, we see that ( + 1 )2 +
1
( + ) 1 = 4 cos2 () + 2 cos() 1 where = 2/5. Thus, 4 cos2 + 2 cos
1 = 0 if and only if 4 cos2 = 1 2 cos . Then, 4 cos2 = 1 2 cos
2(cos(2) + 1) = 1 2 cos 2 cos(2) = 2 cos 1. Now, we can check
1+ 5
to see that this equality is true. In (c), we shall nd that cos(2/5) = .
4
1 5
Using this, we nd that LHS= = RHS.
2
1
(c) We see that + = 2 cos(2/5). In (d), we nd that
cos(2/5) =
(1 + 5)/4, so we must have 2 cos(2/5) = + 1 = (1 + 5)/2.
2
(d) From (b), we have 4 cos + 2 cos 1 = 0, so if x = cos , we have

4x + 2x 1 = 0 which has solutions at x = 14 5 or x = 
2 1+ 5
4 . Hence,

1+ 5 1+ 5
cos = 4 . Since sin2 + cos2 = 1, we know that sin2 = 4 1 =

55
8 . Thus, sin =  10+24
5
.
   h   i
1+ 5 10+2 5 1+ 5
(e) We need to nd Q , : Q . Let us rst nd Q : Q .
4 4 4
 
1+ 5 1+ 5 1+ 5
In Q , we have a polynomial of the form x which has
4 4 4
 
1+ 5 1+ 5
as a root. Thus, in Q , we have an x such that x = . Hence,
4 4
2 2 2
4x + 1 = 5 and 16x + 8x + 1 = 5,so 16xh +8x 4 =  0. Thus,
i 16x + 8x 4
1+ 5 1+ 5
is the minimum polynomial for , so Q : Q = 2.
4
    
4
10+2 5 1+ 5 10+2 5
Let us now nd Q
4 : Q 4 . As above, x =
4
p
4x = 10 + 2 5 16x2 102 5 = 0. Thus, 16x2 102 5 is the minimum
 
10+2 5 1 5
polynomial of over Q .
4 4
         h 
  i
1+ 5 10+2 5 10+2 5 1+ 5 1+ 5
Now, Q , : Q = Q : Q Q : Q =
4 4 4 4 4

2 2 = 4 = 22 . Thus, a regular pentagon is constructible.

4. Prove that a regular heptagon is not constructible.


2
Needless to say, = cos(2/7) + i sin(2/7) = ei 7 is a primitive seventh
i 2 7
root of unity. As above, x = e 7 x 1 = 0. Thus, [Q() : Q] = 7
CHAPTER 6. FIELDS 40

which is not a power of 2. Hence, a regular heptagon is not constructible


since [Q() : Q] = [Q(cos(2/7), sin(2/7)) : Q(cos(2/7))] [Q(cos(2/7) : Q]
and thus [Q(cos(2/7), sin(2/7)) : Q(cos(2/7))] cannot be a power of 2.

6.4 Splitting Fields

1. Determine the splitting elds in C for the following polynomials (over


Q).
(a) x2 2
2
(b) x + 3
4 2
(c) x + x 6
3
(d) x 5

2 2

x 2 = 0 x = 2 x = 2. Thus, the polynomial has all its roots
(a)
in Q( 2).
2 2

(b) x + 3 = 0 x = 3 x = i 3. Thus, the polynomial has all its

roots in Q(i 3).
2 2
(c) Let u = x . Then, u + u + 6 = 0 (u 2)(u + 3) = 0 u = 2 or

u = 3. Thus, x = 2 or x = i 3. Thus, the splitting eld is Q( 2, i 3).
3 3 1/3 1/3 1/3 i 2 4
(d) x 5 = 0 x = 5 x = 5 . Thus, Q(5 , 5 e 3 , 51/3 ei 3 ) is
the splitting eld.

2. Determine the splitting elds in C for the following polynomials (over


Q).
(a) x3 1
4
(b) x 1
3 2
(c) x + 3x + 3x 4

2 4
(a) x3 1 = 0 x3 = 1. Q(ei 3 , ei 3 ) is the splitting eld.
Thus,
4 4 i i 3
(b) x 1 = 0 x = 1. Thus, Q(e 2 , e 2 ) is the splitting eld.
3 2 3
(c) By completing the cube, we nd that x +3x +3x4 = 0 (x+1) = 5.
3 1/3 1/3 i 2 i 4
If y = x+1, then the solutions to y = 5 are 5 , 5 e 3 , e 3 . Thus, the roots
2 4
1/3
of the original polynomial are 1 = 5 1, 2 = 51/3 ei 3 1, 3 = 51/3 ei 3 1.
Thus, a splitting eld for the polynomial would be Q(1 , 2 , 3 ).

3. Determine the splitting elds over Z2 for the follwing polynomials.


(a) x2 + x + 1
2
(b) x + 1
3
(c) x + x + 1
3 2
(d) x + x + 1

 (a) Following Example 6.4.4, identify


 Z2 with GL2 (Z2 ). We see that A=
0 1
is such that A2 + A + I = 0. Hence, A is the companion matrix of
1 1
CHAPTER 6. FIELDS 41

x2 + x + 1 . Thus, the set of matrices of the forma0 I + a1 A where ai Z2 form


2
an extension eld of Z2 in which
 x + x + 1 has a root.
0 1 2
(b) Similarly, let A = . We see that A +I = 0, so A is the companion
1 0
matrix. Thus, {a0 I + a1 A GL2 (Z2 )|a0 , a1 Z2 } is a splitting eld.

0 0 1
(c) Similarly, let A = 1 0 1. We see that A3 + A + I = 0, so A is the
0 1 0
2
companion matrix. Thus, {a0 I + a1 A + a2 A GL2 (Z2 )|a0 , a1 , a2 Z2 } is a
splitting eld.

0 0 1
(d) Let A = 1 0 0. We see that A3 +A2 +I = 0, so A is the companion
0 1 1
2
matrix. Thus, {a0 I + a1 A + a2 A GL2 (Z2 )|a0 , a1 , a2 Z2 } is a splitting eld.

4. Let p be a prime number. Determine the splitting eld for xp 1 over Q.


2 4 2(p1)
We see that xp 1 = 0 xp = 1. Thus, the roots are {1, ei p , ei p , ..., ei p }.
If p = 2, then the solutions are 1, so the splitting eld would be Q. If p 6= 2,
2(p1)
i 2 i
then the splitting eld would be Q(e p , ..., e p ). (None of the roots except
for 1 are from Q because p is prime and so the fraction in the exponent will
never simplify to an expression in Q).

5. Determine the splitting eld for xp x over Zp .

Since p must be prime for Zp to be a eld, by Fermat's little theorem,


xp x(mod p), so xp x 0(mod p) and Zp would be the splitting eld.

6. Determine the splitting eld for x9 x over Z3 .


Q5
We have that x9 x = (x3 )3 x x3 x(mod 3) 0(mod 3). Thus, the
splitting eld is Z3 .

7. Prove that if F is an extension eld of K of degree 2, then F is the


splitting eld over K of some polynomial.

If [F : K] = 2, then F is a vector space over K with some basis {1, u}


for some u F K . Since [F : K] = 2, we know that u2 can be written as
k0 + k1 u for some k0 , k1 K . Hence, u2 k1 u k0 = 0. Thus, the polynomial
x2 k1 x k0 has u as a root and u F K . (The other root must also be in
F , since we can factor out the root to obtain x2 k1 x k0 = (x u)(x r). If
r K , when we foil the expression on the RHS out, we get a polynomial whose
coecients do not come from K . Contradiction.) Thus, F is the splitting eld
2
for x k1 x k0 .
CHAPTER 6. FIELDS 42

8. Let K be a eld. For a monic polynomial f (x) = a0 + a1 x + +


an1 xn1 +xn in K[x], the following matrix C is called the companion matrix
of f (x):
0 1 0 0
0
0 1 0
0 0 0 0
.. .. .. .. .. .

.
. . . .
0 0 0 1
a0 a1 a2 an1
This exercise outlines the proof that f (C) = 0. (That is, a0 I + a1 C + +
an1 C n1 + C n = 0, where I is the n n identity matrix.) Let v1 = (1, 0, ..., 0),
v2 = (0, 1, ..., 0), ... ,vn = (0, 0, ..., 1) be the standard basis vectors
Pn for K .
n

(a) Show that vi C = vi+1 for i = 1, ..., n 1 and vn C = j=1 aj1 vj .


(b) Find similar expressions for v1 C , ..., v1 C , v1 C n , and show that v1 f (C) =
2 n1

0.
(c) Show that vi f (C) = 0, for i = 2, ..., n and conclude that f (C) = 0.


0 1 0 0

0 0 1 0

  0 0 0 0
(a) We see that 0 1 0 . will

. . .. .
.. .
.
.
. . .
.

0 0 0 1
a0 a1 a2
an1
give a 1 n vector. By matrix multiplication, to obtain the (1, k)th entry of the
 
vector, we take the dot product of the rst (and only) row of 0 1 0
with the k th column of C . Thus, the (1, k)th entry will be 0 if the (k, i)th entry
of C is 0. The only nonzero entries of C are the superdiagonal and the bottom
row. So the (1, k)th entry of the product will be 1 if k 1 = i and will be ak
Pn
if i = n. Hence, vi C = vi+1 for i = 1, ..., n 1 and vn C = j=1 aj1 vj .

(b) We can write v1 C 2 = v1 CC = v2 C = v3 . Arguing similarly, we see


m n
Pn v1 C = vm+1
that for 1 m n 1. Then, v1 C = v1 C n1 C = vn C =
j=1 aj1 vj .
n1
Next, v1 f (C) = a0 v1 I + a1 v1 C + + an1 v1 C
Pn + v1 C n = a0 v1 + a1 v2 +
+ an1 vn + j=1 aj1 vj . Note that the sum at the end cancels every term
before it. Thus, v1 f (C) = 0.


(c) Examine vi f (C) = v1 C i1 f (C) = v1 a0 C i1 + a1 C i + + an1 C n1+i + C n+i =
v1 a0 C i1 + v1 an1 C n1+i +v1 C n+i = (v1 a0 I+ +v1 an1 C n1 +v1 C n )C i =
0 C i = 0. Now vi f (C) = 0 for all i 2, so by the axioms for eld, f (C) = 0.

9. Let K be a eld, let f (x) = a0 + a1 x + + an1 xn1 + xn F [x], and let


C be the companion matrix of f (x), as dened in Exercise 8. Show that the set
CHAPTER 6. FIELDS 43

R = {b0 I + b1 C + + bn1 C n1 |bi F for i = 0, ..., n 1} is a commutative


ring isomorphic to the ring F [x]/hf (x)i.

First of all, I believe that there is a typo in this question, as K is introduced


but not used in any way. I assume that K should have been F.
Example 6.4.4 practically establishes the ring properties, so let us prove the
isomorphic claim.
Note that F [x]/hf (x)i can only be a ring if f (x) is irreducible. Dene :
F [x]/hf (x)i (b0 + b1 x + + bn1 xn1 ) = b0 I + b1 C + + bn1 C n1 .
by
n1
(1-1)Recall that by how C was dened, b0 I + b1 C + + bn1 C = 0 if
and only if the coecients of C terms correspond to the coecients of x terms
in f (x). Thus, ker = 0. Thus, is one-to-one.
n1
(Onto)If we wish to obtain a0 I + a1 C + + an1 C , we simply take the
n1
image (a0 I + a1 x + + an1 x ). Thus, is onto.
(Preservation of +)Addition in R is clearly the same as for polynomials, so
addition is preserved.
(Preservation of ) In F [x]/hf (x)i, we have the relation xn = b1n1 x(b0 +
n2
b1 x + + bn2 x ). Since b0 I + b1 C + + bn1 C n1 = 0, we have the
n 1 n2
relation C = bn1 C(b0 + b1 C + + bn2 C ). We have similar relations
n+1 n+2
for x ,x , .... Thus, multiplication in R corresponds to multiplication in
F [x]/hf (x)i.
Thus, F [x]/hf (x)i = R.

10. Strengthen Theorem 6.4.2 by proving under the conditions of the theorem
there exists a splitting eld F for f (x) over K for which [F : K] is a divisor of
n!.

We need to show that [F : K] = n k for some 0 k 1. We procede


by induction on n. Suppose thatn = 1. Then, F = K is a splitting eld and
[F : K] = 1|n!.
Suppose that there exists an F/K such that [F : K]|n! for all n N .
Now, suppose that deg f (x) = N + 1. By Kronecker's Theorem, there is
E/K where r E is a root of f (x). Hence, f (x) = g(x)(x r) over K(r) where
deg g(x) = N . Now, by the inductive hypothesis, there is F/K(r) such that
g(x) splits over F and [F : K(r)] = N k where 0 k < n. Since the minimum
polynomial of r is of degree N +1 at most, we have [K(r) : K] N +1, so we see
that [F : K] = [F : K(r)][K(r) : K] (N k)(N + 1). Thus, [F : K]|(N + 1)!,
as desired. (Recall that if x, y z , we have xy|z!.)

11. Let K be a eld, and let F be an extension eld of K . Let : F F


be an automorphism of F such that (a) = a, for all a K . Show that for any
polynomial f (x) K[x], and any root u F of f (x), the image (u) must be a
root of f (x).

Suppose that f (x) = an xn + +a1 x+a0 has a root at u F . Then, f (u) = 0


or in other words an un + + a1 u + a0 = 0. Then, since is a homomorphism,
CHAPTER 6. FIELDS 44

(0) = 0 and we get 0 = (an un + +a1 u+a0 ) = an (un )+ +a1 (u)+a0 =


an (u)n + + a1 (u) + a0 (since xes all elements of K ). Hence, (u) is a
root.

12. Use Exercise 11 to show that there are only two automorphisms of the
eld Q(i): the identity automorphism and, and the one dened by (a + bi) =
a bi, for all a, b Q.

We must show that any automorphism of Q(i) must x Q in order to use


Q11. Suppose that : Q(i) Q(i) is an  automorphism. Then, (m) =
1 1
(1 + 1 + + 1) = m (1) = m. Also,
1
n = (n ) = [(n)] = n1 = n1 .
m 1
 m
Hence
n = (m) m = n .
The element a + ib 7 a + (i)b, so let us simply consider what the possible
mappings of i are. We know that a minimal polynomial for i is found by
x = i x2 = 1. Thus x2 + 1 is the minimal polynomial of i over Q. We
see that i are the roots to this polynomial. Thus, there are two possible
mappings of i: the identity mapping or mapping to its additive inverse. Thus,
a + ib 7 a + ib or a + ib 7 a ib.

13. Use Exercise 11 to show that there are at most four distinct automor-
phisms of the eld Q( 2, 3).

As proved in Q12, every element of


Q must
be xed by any automorphism

containing Q. Suppose that Q( 2, 3)
: Q( 2, 3) is an automor-

phism. Then, we must have (a + b 2 + c 3 + d 6) = a + b( 2) + c( 3) +

d( 2)( 3). As in Q12,we onlyneed to consider the mappings of 2 and 3.
2 2
Minimal polynomials for
Q arex 2 and x 3, respectively.
2 and 3 over
The roots of these polynomials are 2 and 3, respectively. Thus, by Q11,

2 7 2 and 3 7 3. Thus, there are four possible mappings.

14. Show thatthe splitting eld of x4 2 over Q is Q( 4 2, i).
(a)
(b) Show that Q( 2, i) is also the splitting eld of x + 2 over Q.
4 4

(a) We see that x4 2 = (x + 21/4 i)(x 21/4 i)(x 21/4 )(x + 21/4 ). (These
4 1/4
were obtained by solving x = 2 as discussed in A.5.) Hence, Q(2 , i) is the
splitting eld.
(b) The roots of x4 + 2 are 21/4 + 21/4 i, 21/4 21/4 i, 2

1/4
+ 21/4 i,
1/4 1/4 4
and 2 2 i. All of these elements are contained in Q( 2, i) and since

4
each element needs both i and 2, it is the splitting eld.

15. Use Exercise 11 to show that there are at most eight distinct automor-
phisms of the splitting eld Q( 4 2, i) of x4 2 over Q.

Let : Q( 4 2, i) Q( 4 2, i) be an automorphism. Then, (a + b(21/4 ) +
c(21/2 ) + di + e(21/4 i) + f (21/2 i)) = a + b(21/4 ) + c(21/4 )(21/4 ) + d(i) +
CHAPTER 6. FIELDS 45

e(21/4 )(i) + f (21/4 )(21/4 )(i). A minimal polynomial for 21/4 over Q is
x4 2. By Q11, this means that (21/4 ) can be mapped to any of the four
2
solutions. Now, x + 1 is the minimal polynomial for i and so i 7 1. Thus,
there are 4 2 = 8 possible automorphisms.

6.5 Finite Fields

1. Give addition and multiplication tables for the nite eld GF(23 ), as
described in Example 6.5.3.

Use the relation x8 = x (among others) to construct the multiplication table.


Addition is the same as polynomial addition.
3
Let us examine another way of constructing GF(2 ). We see that x3 + 1 is
3 3
irreducible over Z[x], so Z[x]/hx +x+1i = GF(2 ). Additionally, we previously
found that Z[x]/hx +1i
3
= {a0 I +a1 C +a2 C 2 GL3 (Z2 )|a0 , a1 , a2 Z2 } where
0 1 0
C = 0 0 1 is the companion matrix for x3 + 1. Thus, we can use ordinary
1 0 0
matrix multiplication and addition to nd the desired tables. This may be an
easier option if a computer is available.

2. Give addition and multiplication tables for the nite eld GF(32 ), and
nd a generator for the gyclic group of nonzero elements under multiplication.

The tables are left to the reader. After constructing the multiplication table,
the reader should be able to readily identify the generator. Simply start with
the rst nonzero element (call it x), look up the product of it with itself. Then
take the product and multiply it by x again. Continue the process until (1) all
elements are accounted for or (2) a duplicate is generated. In the case of (2),
move on to the next nonzero element and start the process over.

3. Find a generator for the cyclic group of nonzero elements of GF(24 ).

We see that x4 + x + 1 is irreducible over Z2 [x] and so Z2 [x]/hx4 + x + 1i


=
4
GF(2 ). One can construct the multiplication table of the nonzero elements of
this eld and identify the generator. The generator happens to be x+hx4 +x+1i.

4. Find the splitting elds over GF(3) for the following polynomials.
(a) x4 + 2
4
(b) x 2.

x = 1 is a root, so x4 + 2 = (x 1)(x3 + x2 + x + 1). Then,


(a) We see that
x + x + x + 1 has x = 2 as a root, so x4 + 2 = (x 1)(x 2)(x2 + 1). This is
3 2

completely factored over Z3 , so the splitting eld is Z3 [x]/hx + 1i


2
= GF(32 ).
CHAPTER 6. FIELDS 46

(b) We see that x4 2 is irreducible over Z3 , so the splitting eld is Z3 [x]/hx4


2i
= GF(34 ).

5. Show that x3 x 1 and x3 x + 1 are irreducible over GF(3). Construct


their splitting elds and explicitly exhibit the isomorphism between these elds.

Just plug in x = 0, 1, 2 to both polynomials and observe that the output is


never zero. Thus, both polynomials are irreducible. Thus, Z3 [x]/hx3 x 1i
3 3
is the splitting eld of x x1 and Z3 [x]/hx x + 1i is the splitting eld
of x3 x + 1. Since both elds have the same number of elements (namely 27),
they must be isomorphic.
Dene : Z3 [x]/hx3 x 1i Z3 [x]/hx3 x + 1i dened by (f (x) + hx3
x 1i) = (f (x) 2) + hx3 x + 1i.
(1-1) We have just subtracted 2 from every element, so is one-to-one.
3
(Onto) If we wish to obtain g(x) + hx x + 1i, we use the input g(x) + 2 +
3
hx x 1i. Thus, is onto.
(Preservation of +) Addition is corresponds to polynomial addition, so + is
clearly preserved.
Z[x]/hx3 x1i, we have the relation x3 +hx3 x1i =
(Preservation of ) In
x + 1 + hx x 1i. In Z[x]/hx3 x + 1i, we have the relation x3 + hx3 x + 1i =
3

x 1 + hx3 x + 1i. Since (x + 1 + hx3 x 1i) = x 1 + hx3 x + 1i,


we see that multiplication will correspond. Thus, the two splitting elds are
isomorphic.

6. Show that x3 x2 + 1 is irreducible over GF(3). Construct its splitting


eld and explicitly exhibit the isomorphism between this eld and the splitting
eld of x3 x + 1 over GF(3).

Just plug in x = 0, 1, 2 and observe that the output is never 0. Thus, the
polynomial is irreducible. The splitting eld for the polynomial would then be
Z3 [x]/hx3 x2 + 1i. One can construct multiplication tables for both elds,
identify the generator and map them to each other.

7. Show that if g(x) is irreducible over GF(p) and g(x)|(xp x), then
m

deg(g(x)) is a divisor of m.
m m
We know that xp x has splitting eld GF(p
m
). Now, if g(x)|(xp x),
m
m
then g(x) must split over GF(p ) since xp x splits over it. Suppose that F
m
is the splitting eld of g(x). Then, F is a subeld of GF(p ) since g(x) splits
over it. Also, [F : GF(p)] = deg(g(x)) since g(x) is the minimal polynomial
m
over its splitting eld. By Lemma 6.5.5 and its proof, [GF(p ): GF(p)]= m.
Furthermore, m = [GF(pm ) : GF(p)] = [GF(pm ) : F ][F : GF(p)] = [GF(pm ) :
F ] deg(g(x)). Thus, deg(g(x)) is a divisor of m.

8. Let m, n be positive integers with gcd(m, n) = d. Show that, over any


eld, the greatest common divisor of xm 1 and xn 1 is xd 1.
CHAPTER 6. FIELDS 47

Let m = qd and n = rd. We see by simple polynomial long division that


xdq 1 = (xd 1)(xd(q1) + xd(q2) + + x + 1) and similarly, xrd 1 = (xd
1)(xd(r1) + xd(r2) + + x + 1). We now examine p1 (x) = xd(q1) + + x + 1
d(r1)
and p2 (x) = x + + x + 1 to show that these two factors share no
divisors. Assume without loss of generality that q r . Then, we can write
p1 (x) = p2 (x) + xd(q1) + xd(q2) + + xd(r+1) + xdr . We see clearly that
p1 (x) p2 (x) + xd(q1) + + xr p2 (x) xd(q1) + + xr
= = + . Clearly, the
p2 (x) p2 (x) p2 (x) p2 (x)
second term is not perfectly divisible. (If it were, we'd have p2 (x) as a factor of
xd(q1) + +xr which would imply that p1 (x) = p2 (x)[1+p3 (x)] for some p3 (x)
of degree less than p2 (x). But since p1 (x) is of degree q and p2 (x) is of degree
r such that (q, r) = 1, this is impossible.) Thus, gcd (xm 1, xn 1) = xd 1.

9. If E and F are subelds of GF(pn ) with pe and pf elements respectively,


how many elements does E F contain? Prove your claim.

Since E F are subelds, they have pe and pf elements where e|n and
and
f |n. Then, E F is a subeld of both E and F , so the number of elements
e f gcd(e,f )
must divide both p and p . Thus, E F contains p elements.

10. Let p be an odd prime.


(a) Show that the set S of squares in GF(p ) contains (p + 1)/2 elements.
n n

(b) Given a GF(p ), let T = {a x|x S}. Show that T S 6= .


n

(c) Show that every element of GF(p ) is a sum of two squares.


n

(d) What can be said about GF(2 )?


n

n
(a) We know that multiplication in GF(p ) is cyclic with some generator
2 2k
x. Then x generates x 0 k < p . This element generates (pn 1)/2
for all
n
2 n
elements. Let us not forget that 0 = 0 so 0 S . Thus, |S| = (p + 1)/2.
(b) Suppose a = 0. Then, 0 S , so 0 0 = 0 T and so T S 6= . Also,
if a = 1, then 1 1 = 0 T S .
k k 2`
Otherwise, we have a = x 0 for some xed k0 Z. Then, x 0 x =
k0 2`k0 2` 2`k n
x (1 x ). Thus, choose x such that x = 1. (Since GF(p ) is
cyclic, we can always do this.)
0 = 02 + 02 and 1 = 12 + 02 are sums of squares. Now,
(c) It is clear that
k n k 2`
let x GF(p ). Then, by (b), x x = x2j for some x2` S and j Z+ .
k 2j 2`
Hence, x = x + x = (x ) + (x ) . Thus, any element of GF(pn ) is the sum
j 2 ` 2

of two squares.
n
(d) Suppose that y GF(2 y = a2 + b2 = (a + b)2 . Since
) is such that
n n
GF(2 ) is closed under addition, we have that a + b = c GF(2 ) and that
2
y = c . Hence, y is a sum of squares if and only if y is itself a square. (For
2 2 2
instance, if GF(2 ) = {0, 1, a, b}, then a = b and b = a, and so neither a nor
b can be a sum of squares.)
CHAPTER 6. FIELDS 48

11. Show that xp x + a is irreducible over GF(p) for all nonzero elements
a GF(p).

We know that xp = x for any x GF(p). Hence,xp x + a = x x + a = a.


p
There are no roots to a constant polynomial, so x x + a is irreducible over
GF(p).

12. Dene the function : GF(23 ) GF(23 ) by (x) = x2 , for all x


GF(23 ).
(a) Show that is an isomorphism.
(b) Choose an irreducible polynomial p(x) to represent GF(2 ) as Z2 /hp(x)i,
3

and give an explicit computation of , , and .


2 3

3
(a) (1-1) Here is the multiplication table for GF(2 ):

Note that every element is a square. Thus, the map is one-to-one.


(Onto) Any nite map that is one-to-one is also onto.
(Preservation of +) (a + b) = (a + b)2 = a2 + b2 = (a) + (b). (See Lemma
6.5.4.)
(Preservation of ) (ab) = (ab)2 = a2 b2 = (a)(b).

(b) Let p(x) = x3 + 1. (Check that this has no roots by using the multipli-
cation table above.) Then,Z2 /hp(x)i has 8 elements and is thus isomorphic to
3
GF(2 ). We must have (ax2 + bx + c + p(x))2 = a2 x4 + b2 x2 + c2 + p(x). Since
x3 = 1, we know x4 = x, so (ax2 + bx + c) = bx2 + a2 x + c2 . The rest of the
maps are similar.

6.6 Irreducible Polynomials over Finite Fields

1. Verify Theorem 6.6.1 in the special case of x16 x over GF(2), by


multiplying out the appropriate irreducible polynomials from the list given in the
answer to Exercise 12 of section 4.2.
CHAPTER 6. FIELDS 49

The list of irreducible polynomials is: x, x + 1, x2 + x + 1, x3 + x2 + 1,


x + x + 1, x + x + x + 1, x + x + 1, and x4 + x + 1. Now, by Theorem
3 4 3 2 4 3
24
6.6.1, the monic irreducible factors of x x should be the monic irreducible
2
polynomials of degree 4, 2, and 1. Thus, we must show that x(x + 1)(x + x +
4 3 2 4 3 4 16
1)(x + x + x + x + 1)(x + x + 1)(x + x + 1) = x x.
2 2 2 4 3 2 4
First, x(x+1) = x +x. Next, (x +x)(x +x+1) = x +2x +2x +x = x +x.
4 4 3 2 8 7 6 3 2
Then, (x + x)(x + x + x + x + 1) = x + x + x + x + x + x. Next,
(x8 + x7 + x6 + x3 + x2 + x)(x4 + x3 + 1) = x12 + x9 + x8 + x6 + x4 + x3 + x.
12
Finally, (x + x9 + x8 + x6 + x4 + x3 + x)(x4 + x + 1) = x16 + x = x16 x.

2. Use Theorem 6.6.1 to show that over GF(2) the polynomial x32 +x factors
as a product of the terms x, x + 1, x5 + x2 + 1, x5 + x3 + 1, x5 + x4 + x3 + x + 1,
x5 + x4 + x2 + x + 1, x5 + x3 + x2 + x + 1, and x5 + x4 + x3 + x2 + 1.
5
We have that x32 + x = x2 x, so by Theorem 6.6.1, the irreducible factors
must be the irreducible polynomials of degree 5 and 1. It is left to the reader
to show that the list of factors is complete. To do this, realize that in order for
the polynomial to be irreducible, it must be linear or have an odd number of
terms. Since 5 has no other divisors than 5 and 1, the non-linear polynomials
must be of degree 5 and have an odd number of terms.

3. Let F be a eld of characteristic p, with prime subeld K = GF(p). Show


that if u F is a root of the polynomial f (x) K[x], then up is also a root of
f (x).

Let f (x) = an xn + + a1 x + a0 . If f (u) = 0, then 0 = f (u)p = (an un +


+ a1 u + a0 )p = apn (up )n + + ap1 up + ap0 . (The last equality holds since
p
char(F ) = p.) Now, by Fermat's Little Theorem, ai ai (mod p), so the last
p n p p
expression in the equality becomes an (u ) + + a1 (u ) + a0 = f (u ). Thus,
p
f (u ) = 0.

4. Let u be a primitive element of GF(pm ) and let M (i) (x) be the minimum
polynomial of ui over GF(p). Show that every element of the form uip is also
k

a root of M (i) (x).

u is primitive, then hui = GF(pm ) . By denition


If
 of M
(i)
(x), we have
(i)
i (i) i p (i) ip
M (u ) = 0. By Q3, we know that M (u ) = M (u ) = 0. Similarly,
 2
(i) ip
since M (u ) = 0, we know that M (i) (uip )p = M (i) (uip ) = 0. Continuing
(i) ipk
this argument, we see that M (u ) = 0.

5. Let GF(26 ) be represented by Z2 [x]/hx6 +x+1i, and let u be any primitive


element of GF(26 ). Show that GF(23 ) = {0, 1, u9 , u18 , u27 , u36 , u45 , u54 }.
6
I'm not sure why GF(2 ) would have to have the representation discussed
above. If u is primitive, then hui = GF(26 ) (which has a multiplicative group
CHAPTER 6. FIELDS 50

3 6
structure). Since GF(2 ) is a subeld of GF(2 ) (since 3|6), we know that
3 6 3
GF(2 ) GF(2 ) . By Cauchy's theorem, any generator of GF(2 ) must
3
have order dividing 64. Since |GF(2 )| = 8, the only possible element that can
3
be a generator is u9 . Hence
9 3 9 54
GF(2 ) = hu i. Thus, GF(2 ) = {0, 1, u , ..., u }.

6. Let F be a eld, and let n be a positive integer. An element F is


called a primitive nth root of unity if it has order n in the multiplicative
group F . Show that no eld of characteristic p > 0 contains a primitive pth
root of unity.

If F has characteristic p, then |F | = pk 1 for some k. Now, by clearly,


k
p is not a divisor of p 1. (The division algorithm can be used to show this.)
Thus, by Cauchy's Theorem, there cannot exist an element of F of order p.
Hence, there can be no primitive pth root of unity.

7. Let n Z+ , and dene (n) to be the number of divisors of n.


(a) Show that is a multiplicative function.
(b) Show that if n = p1 p2 pk , then (n) = (1 + 1)(2 + 1) (k + 1).
1 2 k

(c) Show that (n) is odd if and only if n is a square.


(d) Show that d|n (d)(n/d) = 1.
P

(a) Suppose that n has divisors r and s such that (r, s) = 1. Then, r and s
share no factors, so (rs) = (r) (s).
1 2 k i
(b) By (a), we have (n) = (p1 ) (p2 ) (pk ). Then, each pi has
2 i i
1, pi , pi , ..., pi as divisors, so (pi ) = i + 1. Hence, (n) = (1 + 1)(2 +
1) (k + 1), as desired.
(c) If n is a square, then all of its prime factors are raised to an even power,
1 k
so by (b), (n) is odd. If (n) = (p1 pk ) is odd, then by (b) all of the
factors of (1 + 1) (k + 1) must be odd. (For if there were so much as one
i + 1 that were even, then the expression (1 + 1) (k + 1) would be even.)
This implies that i + 1 = 2mi for some mi for all i . Thus, i = 2mi + 1, so
i is odd for all i.
(d) We have established that is a multiplicative function, so if we dene
P
f (n) 1, then (n) = d|n f (d) and so by the Mobius Inversion Formula, we
P
obtain 1 = f (m) = n|m (m/n) (n).

8. Let n Z+ and dene (n) = d|n d, the sum of positive divisors of n.


P

(a) Show that is a multiplicative function.

(b) Show that if n = p1 p2 pk , then (n) = 1)/(pi 1) .


1 2 k Qk i +1

i=1 (pi
(c) Show that (n) is odd if and only if n is a square or two times a square.
(d) Show that n|d (d)(n/d) = n.
P

(a) Suppose that n has two divisors r and s such that (r, s) = 1. Then, r
and s share no divisors, so (rs) = (r)(s). Thus, is multiplicative.
CHAPTER 6. FIELDS 51

Qk
(b) By (a), we see that (n) = i=1 (p
i ).
i
We need to show that (p
i )=
i

p
i
i +1
1
. We do this by induction on i . For i = 1, we get (pi ) = p + 1.
pi 1
2
pi 1 (pi + 1)(pi 1)
Also, = = pi + 1. Thus, the statement is true for
pi 1 pi 1
i = 1. Suppose it is true for all i N . Then, for i = N + 1, we see
N +1 N +1 N +1
that (pi ) = (pN i ) + pi . Thus, by the inductive hypothesis, (pi )=
N +1 N +1 N +2 N +1 N +2
pi 1 p 1 + p p p 1
+pN i
+1
= i = , as required. Hence,
pi 1 pi 1 pi 1
Qk
(n) = i=1 (p i
i +1
1)/(pi 1) .
i
(c) For any prime number pi , we have (pi ) = i + 1. Hence, if i is even,
i i
we will have that (pi ) is a sum of an odd number of terms. Thus, (pi ) is
1 k
odd for all pi if i is even. Hence, if n is a square, (n) = (p1 ) (pk ) is a
product of odd numbers and thus odd.
2k
n is two times a square, then (n) is of the form (n) = (221 +1 )(p2
If 2 ) (pk ).
2

22 2k 21 +1
As previously discovered, (p2 ) (pk ) is odd. Then, (2 ) = 2 + 2.
2 +1
One of the factors is 1, and the rest are even. Thus, (2 1 ) is odd.
i
Conversely, (pi ) is even if i is odd. (See rst paragraph of (c)). Hence,
(n) = (p1 ) (p
1
k ) would be even since one even factor makes the entire
k

product even.

P (d) We have shown to be multiplicative. Let f (n) = n. Then, (n) =


P
d|n f (d). Thus, by the Mobius Inversion Formula, n|d (d)(n/d) = n.

9. A positive integer n is called perfect if it is equal to the sum of its proper


positive divisors. Thus, n is perfect if and only if (n) = 2n. Prove that n is
an even perfect number if and only if n = 2p1 (2p 1), where p and 2p 1 are
prime numbers.

() p and 2p 1 are prime. We show that n = 2p1 (2p 1)


Suppose that
p1
is perfect. Examine (n) = (2 )(2p 1). Since 2p 1 is prime, its only
p p1
divisors are 1 and 2 1. Thus, (n) = (2 ) 2p = (2p 1)2p from Q8. Thus,
p1 p
(n) = 2 2 (2 1) = 2n, as needed.
() Suppose that p is prime, but 2p 1 is not. Then (n) = (2p1 )(2p
1) = (2p 1) (something greater than 2p ). (Since at very least 2p 1 has three
p
factors:1, 2 1 and p , for some prime p. If we add these three up we get
p p
2 + p > 2 .) Thus, (n) > 2n. Thus, n is not perfect.

10. Let D be an integral P domain. Show thatQ if f : Z D is a nonzero


+

multiplicative function, then d|n (d)f (d) = p|n (1 f (p)), for all n Z+ ,
where the product is taken over all prime divisors p of n.
k
Suppose that n has the prime factorization n = p 1 2
1 p2 pk . We form di-
visors ofn by selecting prime factors of n and multiplying them together. Both f
1 2 1 2
and are multiplicative, so we see that (d)f (d) = (q1 q2 qmm )f (q1 q2 qmm ) =
1 1 m m
(q1 )f (q1 ) (qm )f (qm ). Now, if i > 1 for any pi prime factor of a di-
CHAPTER 6. FIELDS 52

visor, then by the denition of , we have that the entire term must equal zero
since (p
i ) = 0.
i
Thus, the only divisors of n that will ever contribute to the
sum are those that are products of prime factors.
P Q
We now show that d|n (d)f (d) = p|n (1 f (p)) by induction on the
number of prime factors of n. Suppose that n = p for some prime p. Then,
P
d|n (d)f (d) = (1)f (1) + (p)f (p) = f (1) f (p) = 1 f (p). (We see that
f (1) = 1 since we must have f (x) = f (1 x) = f (1)f (x).) Thus, we have
established a basis for induction. Let us suppose that this formula holds for M
prime factors. Now, we need to show that it works for M + 1 factors.
1 M +1 M +1 1 M
Let n = p1 pM +1 . Then, n = mpM +1 where m = p1 pM . Then,
P P PM P
d|n (d)f (d) = d|m (d)f (d)f (pM +1 )+ i=1 f (pi pM +1 ) i6=j f (pi pj pM +1 )+
+ (1)M i1 6=i2 6=6=iM f (pi1 pi2 P
P
piM pM +1 ). Since
P f is multiplicative, we
can factor out f (pM +1 ) and obtain d|n (d)f (d) = d|m (d)f (d) f (pM +1 )
 PM 
1 i=1 f (pi ) + i6=j f (pi pj ) + (1)M 1 i1 6=i2 6=iM f (pi1 pi2 piM ) .
P P

We notice that the coecient of


P f (pM +1 ) P
P on the RHS of the equation is equal
P
to d|m (d)f (d). Hence,
d|n (d)f (d) =
Q d|m (d)f (d)f (pM +1 ) d|m (d)f (d).
Q
By the inductive hypothesis, this is equal to
Q  p|m (1f (p))f (pM +1 ) p|m (1
Q
f (p)) = p|m (1 f (p)) (1f (pM +1 )) = p|n (1f (p)), as desired. (Whew!)

11. Let R be a commutative ring. Let R be the set of all functions f : Z+


R. For f, g R, dene f + g by ordinary addition of functions: (f + g)(n) =
f (n) + g(n), for all n Z+ . Dene a product * on R as follows:

f (d)g(n/d), for all n Z+ .


X
(f g)(n) =
d|n

The product * is called the convolution product of the functions f and g .


Dene  : Z+ R by (1) = 1 and (n) = 0, for all n > 1.
(a) Show that R is a commutative ring under the operations + and *, with
identity .
(b) Show that f R has a multiplicative inverse if and only if f (1) is
invertible in R.
(c) Show that if f, g R are multiplicative functions, then so is f g .
(d) Show that if R is an integral domain, then the set of nonzero multiplica-
tive functions in R is a subgroup of R , the group of units of R.
(e) Dene R R as in Denition 6.6.3, with the understanding that 0 and
1 are (respectively) the additive and multiplicative identities of R. Let R be
dened by (n) = 1, for all n Z+ . Show that R = , and that R f = f ,
for all f R.

(a) (Group property of +) Already shown in Ch. 5.


P
f (g+h)(n) = f (g(n)+h(n)) = d|n f (d)(g(n/d)+
(Distributivity) Examine
P P P
h(n/d)) = d|n [f (d)g(n/d)+f (d)h(n/d)]= d|n f (d)g(n/d)+ d|n f (d)h(d/n)=
(f g)(n) + (f h)(n).
CHAPTER 6. FIELDS 53

The other distributive property is shown in a similar way.


P P
(Commutativity) We see that (f g)(n) = d|n f (d)g(n/d) = d|n f (n/d)g(d) =
(g f )(n). (Whatever factors aren't in n/d are in d and vice-versa.)
(b) Suppose that there is g R such that g f = f g = . This happens if
and only if n = 1 implies that 1 = (1) = f (1)g(1) = g(1)f (1). This happens if
and only if f (1) is invertible.
(c) Suppose that f and g are multiplicative functions. Suppose that n = pq
where p and q are prime. Then, (f g)(pq) = f (1)g(pq) + f (p)g(q) + f (q)g(p) +
f (pq)g(1). (1) Next, (f g)(p)(f g)(q) = [f (1)g(p) + f (p)g(1)] [f (1)g(q) +
f (q)g(1)] = g(p)g(q) + g(p)f (q) + f (p)g(q) + f (p)f (q). (2) We see that (1) =
(2) (keeping in mind that for multiplicative functions f and g , we must have
f (1) = g(1) = 1). Hence, f g is multiplicative.

(d) Suppose that f is multiplicative. Then, as always, f (1) = 1 R .

Hence, f R . By (c), the set of multiplicative functions is closed. Thus, the

set of multiplicative functions are a subgroup of R .
1 k 1
(e) Suppose that n = p1 pk . Then, (R )(n) = (R )(p1 ) (R
k Qk Pk i

)(pk ) = j=1 i=0 R (pj ) . Now, clearly (R )(1) = 1. Also, it is clear
that if i > 1 for some i, then (R )(n) = 0. So, suppose that i = 1 for all
Qk
i. Then, we have (R )(n) = j=1 (1 1) = 0. Hence, (R ) = . Now
that this has been established, we have that R f =  f = f (keeping in
mind that the ring R is commutative).

12. Let R be a commutative ring.P Let f : Z R be any


+
function, and let
F : Z R be dened by F (n) = d|n f (d), for all n Z+ . Show that if F is
+

a multiplicative function, then so is f .

Suppose we have (r, s) = 1. Now, by the Mobius Inversion formula, we have


P
f (rs) = n|rs (rs/n)F (n) = 1 F (r) F (s) + F (rs) = 1 F (r) F (s) +
F (r)F (s). (1)
Next, f (r)f (s) = (F (r) + 1)(F (s) + 1) = F (r)F (s) F (r) F (s) + 1.
(2)
We see that (1) = (2), so f is multiplicative.

6.7 Quadratic Reciprocity

    
1. Prove that ab
p = a
p
b
p for all a, b Z such that p - a and p - b.
 
ab
We see that by Euler's Criterion (ab)(p1)/2 a(p1)/2 b(p1)/2 (mod p).
p
      
a
On the other hand,
p a
(p1)/2
(mod p) and pb b(p1)/2 , so ap b
p
    
a(p1)/2 b(p1)/2 (mod p). Thus, abp = ap b
p .

2. Compute the following values of the Legendre symbol.


CHAPTER 6. FIELDS 54

231

(a)
997 
783
(b)
997

231 3711 3 7 11
    
(a) From Q1,
997  = 997 = 997 997 997 . Following Example
3 997
 (9971)(31)/4
6.7.1, we nd that
997 = 3 (1) 9972/2 (1) 498
(mod 3)
3 7 11
 
1(mod 3). Thus,
997 = 1 . Similarly,
997 = 1 and
997 = 1 . Hence,
231

997 = (1)(1)(1) = 1.
 
783 33 29 3
  
(b) Again,
997 =
997 997 . We compute, as above, that 997 = 1.
29 783
 
Also,
997 = 1. Hence, 997 = (1)3 (1) = 1.

3. Is the congruence x2 180873(mod 997) solvable?

First of all, 180873 416(mod 997). To determine if the congruence is


416 2 5 13 2
   
solvable, we must compute
997 = 997 997 . Now, obviously 997 =
9972 1
13
= 1. Hence 416
 
(1) 8 = 1. Also, 997 997 = 1. Thus, the congruence is
not solvable.
 
*4. Determine the value of pr for the indicated values of r, where p is an
odd prime subject to the indicated conditions.
(a) r = 5, p 6= 5
(b) r = 6, p 6= 3
(c) r = 7, p 6= 7
(d) r = 11, p 6= 11
(e) r = 13, p 6= 13
   
5 p p 5
 
(a) We see that
p = 5 (1)4(p1)/4 = 5 . Thus, p p2 (mod 5).
Thus, we have

 
5
Value of p modulo 5
p
1 1
2 -1
3 -1
4 1

p2 1
    
6 3 2 p

(b) We see that
p = p p = 3 (1) 8 . Example 6.7.3 gives the
p

possible values of
3 . Using these, we construct the following table.
 
6
Value of p modulo 12
p
1 1
5 1
7 -1
11 -1
CHAPTER 6. FIELDS 55

 
p
7
(1)6(p1)/4 . Now, p7 p3 (mod 7). If p7 =
  
(c) We see that
p = 7
1, then p 1, 2, or 4(mod 7). If p7 = 1, then p 3, 5, or 6(mod 7). In


addition, we have that p 1 or 3(mod 4). Thus, we create the following table:

 
7
Value of p modulo 28
p
1 -1
9 -1
25 -1
15 1
23 1
11 1
17 -1
5 -1
13 -1
3 1
19 1
27 1

[FINISH]
5. If a is a quadratic nonresidue of each of the odd primes p and q , is the
congruence x2 a(mod pq) sovlable?

Suppose that x2 a(mod pq) has a solution. Then, x2 a = kpq for some
k. This implies that x2 = a + p(kq) = a + q(kp) imlying that x2 a(mod p)
and x2 a(mod q) have solutions. Contradiction.

6-8. [FINISH]
Chapter 7

Structure of Groups
7.1 Isomorphism Theorems: Automorphisms

See other Scribd document.

7.2 Conjugacy

1. Let H be a subgroup of G. Prove that N (H) is a subgroup of G.

Let a, b N (H). Then, we have aHa = H and b1 Hb = H . Then,


1
ab H(ab1 )1 = ab1 Hba1 = a(b1 Hb)a1 = aHa1 = H . Thus, N (H)
G.

2. Let H be a subgroup of the group G. Prove that the subgroups of G


that are conjugate to H are in one-to-one correspondence with the left cosets of
N (H) in G.

LetK H . Then K = aHa1 for some a G. Dene the prescription


1
aHa 7 aN (H). Now, if (aHa1 ) = (bHb1 ), then we have aN (H) =
bN (H) which implies that ab1 N (H). Hence ab1 Hba1 = H . This is
1
equivalent to aHa = bHb1 . Thus, members of G/N (H) are in a one-to-one
correspondence with subgroups of G conjugate to H .

3. Let G be a group with subgroups H and K such that H K . Show that


H is a normal subgroup of K if and only if K N (H).

We see thatH E K if and only if aHa1 = H for all a K . Since any


1
element a such that aHa = H is contained in N (H) by denition, we see that
H E K if and only if K N (H).

56
CHAPTER 7. STRUCTURE OF GROUPS 57

4. Let p be a prime number, and let C be a cyclic subgroup of order p in Sp .


Compute the order of N (C).

The only element of order p in Sp is (1, 2, ..., p). Let K C. Then, every
element of K has the same structure as an element in C by Example 7.2.3. But
since the only element of order p is (1, 2, ..., p), it must be that K = C. Thus,
C E Sp . Thus, |C| = p.

5. Let G be a group, let H be a subgroup of G, and let a G. Show that


there is a subgroup K of G such that K is conjugate to H and aH = Ka.

We must simply show that aHa1 G. Let ah1 a1 , ah2 a1 aHa1 .


Then,
1
(ah1 a )(ah2 a ) 1 1
= (ah1 a1 )(ah1
2 a
1
) = ah1 h2 a1 aHa1 . Thus,
aHa1 G, as desired.

6. Let G be a group, let x, y G and let n Z. Show that y is a conjugate


of xn if and only if y is the nth power of a conjugate of x.

y = (gxg 1 )n y = (gxg 1 )(gxg 1 ) (gxg 1 ) y = gxn g 1 y xn .

7. Find the conjugate subgroups of D4 .

First, all normal subgroups are self-conjugate, and since each subgroup of or-
der 4 has index 2, the subgroups {e, a2 , b, a2 b}, {e, a, a2 , a3 }, and {e, a2 , ab, a3 b}
are all self conjugate.
Next {e, b} {e, a2 } {e, a2 b} (from simple verication) and (ab)a3 b(ba3 ) =
2 3 3
a ba , so {e, ab} and {e, a b} are self-conjugate.

8. Find the conjugacy classes of D5 .

First, D5 = {e, a, a2 , a3 , a4 , b, ab, a2 b, a3 b, a4 b}. The cyclic subgroup {e, a, a2 , a3 , a4 }


has order 5 and is of index 2 and thus normal. Thus, it is its own conjugacy
class. There can be no more subgroups of order 5 (for if there were, there would
be more than ten elements). The only possible orders of subgroups would be
order 2. There are ve such subgroups: {e, ab}, {e, a2 b}, {e, a3 b}, {e, a4 b}, and
{e, b}. We multiply {e, ab} on the left and right by the non-identity elements in
the subgroups above and see that all ve subgroups are conjugate.

9. Describe the conjugacy classes of S5 by listing the types of elements and


the number of each type in each class.
CHAPTER 7. STRUCTURE OF GROUPS 58

Class Calculation of Cardinality Cardinality

[(1)] The identity mapping is unique 1


 
5
[(1,2)] 10
  2 
5 3 1
[(1,2)(3,4)] 15
2 2 2
5
[(1,2,3)] 2! 20
3
[(1,2,3)(4,5)] Same 20
 
5
[(1,2,3,4)] 3! 30
4
[(1,2,3,4,5)] 4! 24
Total: 120

10. Find the conjugacy classes of A4 .

A4 consists of the identity element (1), all 3-cycles (of which there are 8), and
three single products of disjoint transpositions (those being (1,2)(3,4), (1,3)(2,4),
and (1,4)(3,2)) . Since disjoint transpositions commute, we know that the
products of disjoint transpositions cannot be conjugate to any 3-cycle or the
identity. We now nd the conjugacy classes: (1,3)(2,4)[(1,2)(3,4)](1,3)(2,4) =
(1,3)(4,2)(2,1)(4,3)(3,1)(2,4) = (1,2,4)(2,4)(1,3)(1,3,4) = (2,1)(4,3) (1,4)(3,2)[(1,2)(3,4)](1,4)(3,2)
= (3,2)(4,1)(1,2)(3,4)(4,1)(3,2) = (3,2)(2,1,4)(1,4,3)(3,2) = (2,4)(1,3) Thus, (1,2)(3,4)
(1,3)(2,4) (1,4)(3,2). Doing a similar analysis on the 3-cycles, we get that
(1,2,3) (4,3,2) (3,4,1) (2,1,4) and (1,3,2) (4,2,3) (3,1,4) (2,4,1).

11. Find the conjugacy classes for the quaternion group Q dened in Ex-
ample 3.3.7.

We see iji1 = j; kjk1 = j. Thus, j is its own conjugacy class.


Then, jij1 = i = kik1 . Thus, i i.
Then, iki1 = k = jkj1 . Thus, k k. Then, 1 are their own
conjugacy classes.

12. Write out the conjugacy class equations for the following groups.
(a) A4
(b) S5
P
(a) Using Q4, clearly |A4 | = |Z(A4 )| + P[A4 : C(x)] = 1 + (3 + 4 + 4) = 12.
(b) Using Q9, clearly |S4 | = |Z(S5 )| + [S5 : C(x)] = 1 + (10 + 15 + 20 +
20 + 30) = 120.

13. Let the dihedral group Dn be given by elements a of order n and b of


order 2, where ba = a1 b. Show that am is conjugate to only itself and am ,
and that am b is conjugate to am+2k b for any integer k.
CHAPTER 7. STRUCTURE OF GROUPS 59

x am in Dn . We see that x cannot have a factor of b in it


Suppose that
` m 1
since if a b = ya y , then whether y has a factor of b in it or not, the b's in
y and y 1 will cancel. Thus, x must be a power of a. Thus, a` = ak bam bank
1
for some k . By the fact that ba = a b, we see that this implies that a` =
k nm nk
a a a = ak+nm+nk
= a 2nm
= am . Of course, every element is
conjugate to itself as well.
We see that ak bam bbank = ak bam+nk = ak+nm+k = a2k+m b.

14. Show that the Frobenius group F20 (dened in Exercise 12 of Section 7.1)
is isomorphic to the subgroup of S5 generated by the permutations (1, 2, 3, 4, 5)
and (2, 3, 5, 4). Use this fact to help in nding the conjugacy classes of F20 , and
its conjugacy class equation.

The fact that h(1, 2, 3, 4, 5), (2, 3, 5, 4)icomes from the work done in Q13 of
Section 7.1, where we found elements a of order 5 and b of order 4 and that any
element of F20 could be uniquely expressed as a product of these two elements
(i.e., they generate F20 ).
From Q12 of 7.1, Z(F20 ) = 4. Now,
h(1, 2, 3, 4, 5)i = {(1), (1, 2, 3, 4, 5)), (1, 3, 5, 2, 4), (1, 4, 2, 5, 3), (1, 5, 4, 3, 2)} and
h(2, 3, 5, 4)i = {(1), (2, 3, 5, 4), (2, 5)(3, 4), (2, 4, 5, 3)}. So, it is easliy seen that
h(1, 2, 3, 4, 5), (2, 3, 5, 4)i = {(1), (1, 2, 3, 4, 5), (1, 3, 5, 2, 4), (1, 4, 2, 5, 3), (1, 5, 4, 3, 2),(2, 3, 5, 4), (2, 5)(3, 4), (2, 4, 5,
2
Thus, the conjugacy classes of F20 are [a], [b], and [b ]. The class equation
would then be |F20 | = 4 + 4 + 10 + 2 = 20.

15. Show that if a group G has an element a which has precisely two conju-
gates, then G has a nontrivial proper normal subgroup.

Suppose that [a] has precisely two elements. Then, [G : C(a)] = 2 and
C(a) E G (since subgroups of index two are normal.)

*16. Show that for each prime p, there exists a nonabelian group of order
p3 .

Let us examine the possible centers of G. By Lagrange's theorem and Burn-


side's theorem, |Z(G)| = p, or p2 . Any group of order p is cyclic (isomorphic
to Zp ), and the abelian groups of order p2 (recall that all groups of order p2
are abelian) are Z p2 and Zp Zp . The following conclusions are made from the
G/Z(G) theorem.

Z(G) G/Z(G) Conclusion

Zp Zp2 G is abelian
Zp Zp ???
Z p2 Zp G is abelian
CHAPTER 7. STRUCTURE OF GROUPS 60

Thus, we seek to nd a nonabelian group of order p3 with center Z(G)


= Zp
and G/Z(G)
= Zp Zp . [FINISH]

17. Let G be a nonabelian group of order p3 , for some prime number p.


Show that Z(G) must have order p.

The center of any p- group is nontrivial by Burnsides Theorem, so the pos-


|Z(G)| are p, p2 , or p3 . However, if |G| = p2 , then G would be
sibilities for
3
abelian. Since G is given to be nonabelian (so its center cannot have order p ),
we must have |Z(G)| = p.

18. Determine the conjugacy classes of the alternating group A5 and use
this information to show that A5 is a simple group.

We have already found the conjugacy classes for A5 in Q9.

Class Order

[(1)] 1
[(1,2)(3,4)] 15
[(1,2,3)] 20
[(1,2,3,4,5)] 24

We know that a normal subgroup of A5 (or any group, for that matter) is
the union of conjugacy classes. In the following table, for every row, construct
a union of conjugacy classes corresponding to the column headers who have X's
in their cell. The order of this union is given in the right-most column.

[(1)] [(1,2)(3,4)] [(1,2,3)] [(1,2,3,4,5)] Order of union

X 1
X X 16
X X 21
X X 25
X X X 36
X X X 40
X X X 45
X X X X 60

Now, by Lagrange's theorem, the order of any group must divide |A5 | =
60. The only possible unions that have orders dividing 60 are [(1)] = (1) and
[(1)] [(1, 2)(3, 4)] [(1, 2, 3)] [(1, 2, 3, 4, 5)] = A5 . Hence, A5 has no proper
nontrivial normal subgroups. Thus, A5 is simple.

Вам также может понравиться